Sei sulla pagina 1di 81

Full-Length Test 5

06/27/2003

12:16 PM

Page 1

Physical Sciences
Time: 100 Minutes Questions 177

DO NOT BEGIN THIS SECTION UNTIL YOU ARE TOLD TO DO SO.

Full-Length Test 5

06/27/2003

12:16 PM

Page 2

PHYSICAL SCIENCES
DIRECTIONS: Most of the questions in the Physical Sciences test are organized into groups, with a descriptive passage preceding each group of questions. Study the passage, then select the single best answer to each question in the group. Some of the questions are not based on a descriptive passage; you must also select the best answer to these questions. If you are unsure of the best answer, eliminate the choices that you know are incorrect, then select an answer from the choices that remain. Indicate your selection by blackening the corresponding circle on your answer sheet. A periodic table is provided below for your use with the questions.
PERIODIC TABLE OF THE ELEMENTS
1 H 1.0 3 Li 6.9 11 Na 23.0 19 K 39.1 37 Rb 85.5 55 Cs 132.9 87 Fr (223) 4 Be 9.0 12 Mg 24.3 20 Ca 40.1 38 Sr 87.6 56 Ba 137.3 88 Ra 226.0 21 Sc 45.0 39 Y 88.9 57 La * 138.9 89 Ac 227.0 22 Ti 47.9 40 Zr 91.2 72 Hf 178.5 104 Rf (261) 58 Ce 140.1 90 Th 232.0 23 V 50.9 41 Nb 92.9 73 Ta 180.9 105 Ha (262) 59 Pr 140.9 91 Pa (231) 24 Cr 52.0 42 Mo 95.9 74 W 183.9 106 Unh (263) 60 Nd 144.2 92 U 238.0 25 Mn 54.9 43 Tc (98) 75 Re 186.2 107 Uns (262) 61 Pm (145) 93 Np (237) 26 Fe 55.8 44 Ru 101.1 76 Os 190.2 108 Uno (265) 62 Sm 150.4 94 Pu (244) 27 Co 58.9 45 Rh 102.9 77 Ir 192.2 109 Une (267) 63 Eu 152.0 95 Am (243) 64 G d 157.3 96 Cm (247) 65 T b 158.9 97 Bk (247) 66 Dy 162.5 98 Cf (251) 67 Ho 164.9 99 Es (252) 68 Er 167.3 100 Fm (257) 69 Tm 168.9 101 Md (258) 70 Y b 173.0 102 No (259) 71 Lu 175.0 103 Lr (260) 28 Ni 58.7 46 Pd 106.4 78 Pt 195.1 29 Cu 63.5 47 Ag 107.9 79 Au 197.0 30 Zn 65.4 48 Cd 112.4 80 Hg 200.6 5 B 10.8 13 Al 27.0 31 Ga 69.7 49 In 114.8 81 Tl 204.4 6 C 12.0 14 Si 28.1 32 Ge 72.6 50 Sn 118.7 82 Pb 207.2 7 N 14.0 15 P 31.0 33 As 74.9 51 Sb 121.8 83 Bi 209.0 8 O 16.0 16 S 32.1 34 Se 79.0 52 Te 127.6 84 Po (209) 9 F 19.0 17 Cl 35.5 35 Br 79.9 53 I 126.9 85 At (210) 2 He 4.0 10 Ne 20.2 18 Ar 39.9 36 K r 83.8 54 Xe 131.3 86 Rn (222)

GO ON TO THE NEXT PAGE. 2

Full-Length Test 5

06/27/2003

12:16 PM

Page 3

Passage I (Questions 15) An object totally immersed in water is subject to two opposing forces which act vertically. First, the objects weight, which is a result of the Earths gravitational attraction, pulls it downward. Second, there is a buoyant force exerted by the water on the object, which pushes the object upward. Archimedes principle states that the upward buoyant force exerted by the water on the object is equal in magnitude to the weight of the water displaced. A researcher constructs an apparatus, shown in Figure 1, to investigate the forces on inflatable devices used by scuba divers. A heavy, elastic balloon filled with an ideal gas is immersed in a pool of water. At depth d1, the balloon is in equilibrium. In other words, the weight of the balloon is exactly equal to the buoyant force exerted by the water on the balloon. Because the net force is zero, the balloon is suspended motionless in water. Since the balloon is elastic and is filled with an ideal gas, its volume will vary with pressure and temperature. This effect is significant because the pressure at all points in the pool is not a constant. (Note: Assume that the temperature at all points in the pool is constant.)

2. If the water is replaced by a denser fluid, then which of the following will NOT occur? A. B. C. D. The pressure at depth d1 will increase. The density of the balloon will remain the same. The volume of the balloon will decrease. The weight of the balloon will remain the same.

3. A second object besides the balloon is suspended motionless when released at depth d1. What can be concluded? A. B. C. D. Its mass is equal to that of the balloon. Its volume is equal to that of the balloon. Its density is equal to that of the balloon. The buoyant force on it is equal to the buoyant force on the balloon.

4. The balloon is moved to depth d2 and then released. Which way does the balloon go? A. B. C. Upwards, because the weight is greater than the buoyant force. Downwards, because the buoyant force is greater than the weight. Downwards, because the weight is greater than the buoyant force. The balloon does not move, because the weight is equal to the buoyant force.

d1 d2 balloon

D.

water

Figure 1 Balloon of ideal gas immersed in a pool of water

5. An iron ball with a volume of 4 105 m3 is released from a height of 0.35 m above the bottom of a 5-meter deep pool of water. How long does it take the ball to reach the bottom? (Note: The density of iron is 7.9 103 kg/m3, and the density of water is 1.00 103 kg/m3. Assume the viscosity of the water is negligible.) A. B. C. D. 0.03 s 0.3 s 30 s 300 s

1. If the volume of the balloon at depth d1 is doubled and the mass is unchanged, which of the following will also double? A. B. C. D. The weight of the balloon The density of the balloon The density of the water that the balloon displaces The buoyant force on the balloon GO ON TO THE NEXT PAGE. 3

Full-Length Test 5

06/27/2003

12:16 PM

Page 4

Passage II (Questions 612) Although water is a popular inorganic solvent, other solvents are often used for their particular properties. One such solvent is liquid ammonia. Pure ammonia has a freezing point of 77C and a boiling point of 33.35C. It exists as a gas at room temperature. However, since ammonia can be liquefied under pressure, it is sold in cylinders that dispense ammonia as a liquid. Liquid ammonia, like water, undergoes self-ionization (Reaction 1). 2NH3(l) NH4+(am) + NH2(am) K50C = 1030

6. Which of the following is a TRUE statement? A. B. C. D. The amount of undissociated acetic acid is larger in solution 4 than in solution 3. The freezing-point depression of solution 3 and solution 4 are equal. The freezing-point depression of solution 3 is larger than that of solution 4. The freezing-point depression of solution 3 is smaller than that of solution 4.

Reaction 1 (Note: am = ammoniacal solution) Even though liquid ammonia and water are both used to dissolve ionic compounds, these compounds can exhibit different behavior in each solvent. For example, ammonia is a stronger base than water; therefore, weak acids that do not completely ionize in water may do so in liquid ammonia. Another property of liquid ammonia that differs from water is its complexing ability: The silver halide salts, which are insoluble in water, are very soluble in liquid ammonia (Reaction 2). AgX(s) + xNH3(l) Ag(NH3)x+(am) + X (am)

7. What is the effect on the freezing point and boiling point if 3 moles of solid silver nitrate are added to solution 2? A. B. C. D. The boiling point increases and the freezing point decreases. Neither the boiling point nor the freezing point changes. The boiling point decreases and the freezing point increases. Both the boiling point and the freezing point decrease.

8. Ammonium chloride is added to the liquid ammonia solution. As a result, the equilibrium of Reaction 1: A. B. C. D. will shift to the left. will shift to the right. will not be affected. will be reached faster.

Reaction 2 A student prepared four solutions as follows: Solution 1: Solution 2: Solution 3: Solution 4: 2 moles of CaCl2 dissolved in 1500 g of liquid ammonia. 2 moles of CaCl2 dissolved in 1500 g of water. 2 moles of CH3COOH dissolved in 2000 g of water. 2 moles of CH3COOH dissolved in 2000 g of liquid ammonia.

(Note: The freezing point depression constant, Kf, of NH3 is 0.957C/m; that of water is 1.86C/m; the boiling point elevation constant, Kb, of water is 0.52C/m.)

GO ON TO THE NEXT PAGE. 4

Full-Length Test 5

06/27/2003

12:16 PM

Page 5

9. Which of the following represents the volume of gas formed (in liters) at STP when 11 cm3 of liquid ammonia is completely vaporized? (Note: the density of liquid ammonia is 0.682 g/cm3.) A. B. C. D. (0.682 11/17) 22.4 0.682 11/17 (0.682 22.4)/(11 17) (0.682 17)/(22.4 11)

11. What is the equilibrium constant for Reaction 2? A. B. C. D. [Ag(NH3)x+][X] [Ag(NH3)x+] [NH3]x [AgX][NH3]x [Ag(NH3)x+][X] [Ag(NH3)x+][X] [NH3]x

10. The student prepared another solution containing 1500 g of water and an unknown amount of calcium chloride. The boiling point increased to 101.56C. What is the mass of calcium chloride dissolved in this sample? A. B. C. D. 0.167 g 1.67 g 16.7 g 167 g

12. Which of the following accurately describes the action of ammonia in Reaction 2? A. B. C. D. It acts as a Brnsted acid. It acts as a Brnsted base. It acts as a Lewis acid. It acts as a Lewis base.

GO ON TO THE NEXT PAGE. 5

Full-Length Test 5

06/27/2003

12:16 PM

Page 6

Passage III (Questions 1317) The magnitude of the magnetic field at the center of a circular loop is given by B = 0i/2r, where i is the current through the wire, r is the radius of the loop, and 0 is a constant. Figure 1 shows an electric circuit, lying in the plane of the page, designed to generate a magnetic field. A voltage source is connected in series to a switch, a 3- resistor, a circular wire loop, and a 1- resistor. A 2- resistor is connected in parallel with the 1- resistor. The switch, shown in the open position, prevents the flow of current when it is in the open position. Two experiments are conducted with the apparatus. Experiment 1 The voltage source is set to provide 10 V. At time t = 0, the switch is closed. The magnitude and direction of the magnetic field generated by the circular loop are measured at points P and Q. Experiment 2 The switch is closed at time t = 0, and the voltage source provides a voltage which varies in time. Figure 2 shows the graph of this voltage. The magnitude of the magnetic field at point P is measured as a function of time.
voltage source + switch R1 = 3 P r Q

13. In Experiment 1, which of the following would decrease the magnetic field at point P? I. II. III. IV. A. B. C. D. Increasing R1 Increasing R2 Increasing R3 Decreasing r

I only I and IV only I, II, and III only I, II, III and IV

14. In Experiment 1, after the switch is closed, what is the orientation of the magnetic field at P and Q, respectively? A. B. C. D. Into the page, out of the page Out of the page, into the page Into the page, into the page To the left, to the right

15. If the current through R1 is 2.7 A, what is the current through R2? A. B. C. D. 0.9 A 1.8 A 2.7 A 8.1 A

R2 = 3 R3 = 3

Figure 1 Electric circuit designed to generate a magnetic field

Voltage

Time

Figure 2 Output voltage of voltage source as a function of time

GO ON TO THE NEXT PAGE. 6

Full-Length Test 5

06/27/2003

12:16 PM

Page 7

16. In Experiment 2, which of the following graphs represents the magnitude of the magnetic field at point P as a function of time? A. C.

Time

B Time

B.

D.

B
Time

Time

17. The instantaneous power dissipated by the resistors in Experiment 2 is shown below as a function of time. What quantity is represented by the area under the curve?

Power Time

A. B. C. D.

Energy Current Resistance Voltage

GO ON TO THE NEXT PAGE. 7

Full-Length Test 5

06/27/2003

12:16 PM

Page 8

Passage IV (Questions 1823) One of the most important uses of lead today is in the production of automobile batteries. These batteries consist of six lead-acid cells connected in series, and each cell generates approximately 2V during discharge. The leadacid cell is an example of a secondary cellan electrochemical cell that has to be charged by some other electrical source before it can be used. A secondary cell, therefore, acts as an electrolytic cell when charged and a galvanic cell during discharge. Lead-acid cells contain a number of lead (II) sulfate covered grids or electrodes immersed in an aqueous solution of sulfuric acid. These grids have a large surface area so that a large current can be generated rather quickly, i.e., when the cells discharge and the car engine is started. When the cell is charged, lead (II) sulfate undergoes the following two reactions: it is reduced to form lead at the cathode, and it is oxidized to form lead (IV) oxide at the anode. When the cell discharges, the reverse reactions occur; that is, lead is oxidized at the anode to form lead (II) sulfate (Reaction 1) and lead (IV) oxide is reduced at the cathode to form lead (II) sulfate (Reaction 2). Pb(s) + SO42(aq) PbSO4(s) + 2e Reaction 1 PbO2(s) + SO42(aq) + 4H+(aq) + 2e PbSO4(s) + 2H2O(l) Eo = +1.69V Reaction 2 The lead-acid cell should only be charged up to the point where all the lead (II) sulfate is consumed; beyond this point, water can undergo electrolysis to generate gaseous oxygen and hydrogen, both of which can react with the newly formed deposits of lead and lead (IV) oxide. Conversely, the cell should not be stored for long periods of time, or allowed to discharge to the point where there is more than a 30% conversion of lead and lead (IV) oxide since lead sulfate can obstruct the flow of sulfuric acid and damage the lead grids. Eo = +0.36 V

18. When a car engine was started for 3 seconds, a current of 333 amps was generated by the battery. How many grams of lead are used up during this time? [Note: Faradays constant = 9.6485 A. B. C. D. 0.0104 g 0.104 g 1.04 g 10.4 g 104 C/mol]

19. What would be the effect on the discharge voltage if a small amount of lead sulfate was added to a cell immediately after it had been charged? A. B. C. D. The voltage would not change. The voltage would decrease. The voltage would increase. The voltage could not be measured.

20. After six months, the output voltage of a car battery dropped from 12V to 6V. What is the most likely explanation for this observation? A. B. C. D. There is less than 30% conversion of lead and lead (IV) oxide to lead sulfate. Half of the lead-acid cells give no output voltage. The level of aqueous sulfuric acid in the cells is too low. There is 100% conversion of lead and lead (IV) oxide to lead (II) sulfate in all the cells.

GO ON TO THE NEXT PAGE. 8

Full-Length Test 5

06/27/2003

12:16 PM

Page 9

21. Which of the following represents the anode halfreaction when the lead-acid battery is overcharged? A. B. C. D. 2H2O(l) O2(g) + 4H+(aq) + 4e 4H+(aq) + 4e 2H2(g) 2H2O(l) 2H2(g) + O2(g) 2H2O(l) + 2e H2(g) + 2OH(aq)

22. Which of the following procedures would be the most useful in preventing the loss of capacity of a battery while in storage? A. B. C. D. The addition of lead sulfate to the electrolyte Sealing the electrolyte in the battery to prevent gas loss The addition of sulfuric acid to the electrolyte The addition of lead and lead (IV) oxide to the electrolyte

23. What is the balanced equation for the discharge reaction in the lead-acid battery? A. B. C. D. 2PbSO4(s) + 2H2O(l) Pb(s) + PbO2(s) + 4H+(aq) + 2SO42(aq) Pb(s) + PbO2(s) + 2SO42(aq) 2PbSO4(s) + O2(g) 2(aq) 2PbSO (s) 2Pb(s) + 2SO4 4 Pb(s) + PbO2(s) + 4H+(aq) + 2SO42(aq) 2PbSO4(s) + 2H2O(l)

GO ON TO THE NEXT PAGE. 9

Full-Length Test 5

06/27/2003

12:16 PM

Page 10

Questions 24 through 28 are NOT based on a descriptive passage.


24. A steel flagpole is found to be 20 meters high on a day when the temperature is 25C. How would the length of the flagpole change if the temperature drops to 5C? (Note: The coefficient of linear expansion for steel is 11 106 K1.) A. B. C. D. It would increase by 6.6 mm. It would increase by 0.33 mm. It would decrease by 0.33 mm. It would decrease by 6.6 mm.

27. Aluminum oxide readily undergoes the following reactions: Al(OH)3(s) + 3H+(aq) Al3+(aq) + 3H2O(l) Al(OH)3(s) + OH(aq) Al(OH)4(aq) Based on these reactions, aluminum oxide can be classed as: A. B. C. D. an amphoteric compound. an acidic compound. a basic compound. a neutral compound.

25. What is the oxidation state of chlorine in ClO4? A. B. C. D. 1 0 +7 +9

28. If a flask containing 0.28 g of nitrogen and 0.64 g of oxygen has a total pressure of 3 atm, what is the partial pressure of nitrogen in the flask? A. B. C. D. 0.5 atm 1.0 atm 2.0 atm 3.0 atm

26

How many moles of Ba2+ are found in a 500 mL solution of barium sulfate? (Note: The Ksp of BaSO4 = 1.1 1010.) A. B. C. D. 1 106 5 106 1 105 5 105

GO ON TO THE NEXT PAGE. 10

Full-Length Test 5

06/27/2003

12:16 PM

Page 11

Passage V (Questions 2933) The collision theory states that molecules must collide with each other before they can react; the rate at which they collide and react determines the rate of a reaction. If the reactant molecules possess enough energy, they will combine to form an activated complexa very short-lived specieswhich dissociates into product molecules. Consistent with the collision theory, two different mechanisms have been proposed to account for the reaction of iodine with hydrogen. In the first mechanism, molecular hydrogen and molecular iodine collide to form an activated complex. The activated complex then dissociates into two molecules of hydrogen iodide (Figure 1).

In this case, the formation of the activated complex results from the collision of three species, so the reaction is classed as termolecular. The proposed kinetics for this process are as follows: k1 I2(g) k2 2I(g) k3 2HI(g) (fast) (slow)

2I(g) + H2(g)

rate of reaction = k3[I]2[H2] Mechanism 2 In the termolecular reaction, the overall rate of reaction is dependent on the concentration of molecular hydrogen and atomic iodinean unstable intermediate. Since the concentration of atomic iodine cannot be measured, we make the approximation (known as the steady-state approximation) that the rate of production and consumption of atomic iodine is the same, so its concentration does not change. Therefore: k1[I2] = k2[I]2 and rearranging this equation gives: k [I]2 = 1 [I2] k2 Equation 1 Since the concentration of molecular iodine can be measured, substituting Equation 1 into the reaction equation in mechanism 2 gives: rate of reaction = k1k3 [I2][H2] k2

I2

H2

Activated Complex

2HI

Figure 1 First proposed mechanism for the formation of hydrogen iodide Since the formation of the activated complex results from the collision between two molecules (hydrogen and iodine), the reaction is classed as bimolecular. The overall rate of the reaction depends on the concentration of hydrogen and iodine: H2 (g) + I2 (g) k 2HI(g)

rate of reaction = k[H2][I2] Mechanism 1 However, recent experimental investigation suggests that the reaction mechanism is more complex. A second mechanism has been proposed: Initially, iodine dissociates into two iodine atoms; these atoms then react with molecular hydrogen to form two molecules of hydrogen iodide (Figure 2).

Equation 2 The second mechanism, therefore, is similar to the first in that the rate of reaction is dependent on the concentration of iodine and hydrogen. To investigate these two mechanisms, a chemist obtained the following data: Initial H2 concentration (mol/L) Initial I2 concentration (mol/L) 0.25 0.50 0.50 Table 1 Initial rate of formation of HI (mol/Ls) 7.5 1.5 3.0 106 105 105

I2

2I

H2

Activated Complex

2HI

0.25 0.25 0.50

Figure 2 Second proposed mechanism for the formation of hydrogen iodide GO ON TO THE NEXT PAGE. 11

Full-Length Test 5

06/27/2003

12:16 PM

Page 12

29. Which of the following statements is true? A. B. C. D. The data obtained by the chemist supports mechanism 1, but not mechanism 2. The data obtained by the chemist supports mechanism 2, but not mechanism 1. The data obtained by the chemist supports mechanism 1 and mechanism 2. The data obtained by the chemist supports neither mechanism 1 nor mechanism 2.

32. In mechanism 2, the reaction is second order with respect to: A. B. C. D. atomic iodine and hydrogen. atomic iodine. molecular iodine. hydrogen.

30. The change in enthalpy of the slow step in mechanism 2 is 33 kcal/mol K, and the change in entropy is 2 kcal/mol. Is this step spontaneous? A. B. C. D. No, because G is positive. No, because hydrogen iodide is an unstable molecule. Yes, because G is negative. Cannot be determined without more information

33. What is the enthalpy of formation, Hf, of hydrogen iodide at 298 K? (Note: The bond energies of H2(g), I2(g), and HI(g) at 298 K are 436 kJ/mol, 151 kJ/mol, and 298 kJ/mol, respectively.) A. B. C. D. 9 kJ/mol 4.5 kJ/mol 4.5 kJ/mol 9 kJ/mol

31. Assuming mechanism 1 is correct, what is the rate constant, k, for the reaction? A. B. C. D. 7.5 6.0 1.2 2.4 106 L/mols 105 L/mols 104 L/mols 104 L/mols

GO ON TO THE NEXT PAGE. 12

Full-Length Test 5

06/27/2003

12:16 PM

Page 13

Passage VI (Questions 3438) An apparatus known as Kundts tube is set up to measure the speed of sound in aluminum. A schematic of the apparatus is shown in Figure 1. A glass tube is laid horizontally with cork dust spread over the bottom. One end of the tube is closed, while the other is attached via a washer to an aluminum rod clamped at the center. A rag covered with violin rosin is used to stroke the rod, setting up longitudinal vibrations in the aluminum rod. The glass tube is then moved back and forth so that the length of the air column can sustain standing waves. The cork dust is seen to clump at the nodes. The dust clusters are found to be about 5.5 cm apart. This value can be used in conjunction with the speed of sound in air (obtained from literature) to determine the frequency of the vibrations in the air, and hence in the aluminum rod. The speed of sound in aluminum can then be determined using the fact that a node occurs in the middle of the rod (where it is clamped) and antinodes occur at the ends.

36. What is the wavelength of the fundamental vibration in the aluminum rod, if the length of the rod is L? A. B. C. D. 4L L 1/2 L 2L

37. The standing wave sustained in the glass tube is: A. B. C. D. mechanical and transverse. mechanical and longitudinal. electromagnetic and transverse. electromagnetic and longitudinal.

38. Which of the following should be true of the Kundts tube setup? A. B. The tube should be evacuated. The tube should be sealed with air at an initial pressure of 1 atm; no air must be exchanged with the environment. The tube should be sealed with air at an initial density equal to that of aluminum; no air must be exchanged with the environment. The tube should allow air to enter and exit freely somewhere.

aluminum rod
C.

clamp

cork dust collects at nodes

D.

Figure 1 Kundts tube apparatus 34. The speed of sound in an ideal, diatomic gas is given by v = 1.40P/, where P is the pressure and is the density. In which of the following gases is the speed of sound the fastest? A. B. C. D. Hydrogen at 20C Hydrogen at 0C Air at 20C Air at 0C

35. What is the frequency of the vibrations in the rod? (Speed of sound in air = 344 m/s) A. B. C. D. Cannot be determined without the length of the rod Cannot be determined without the length of the tube 6200 Hz 3100 Hz

GO ON TO THE NEXT PAGE. 13

Full-Length Test 5

06/27/2003

12:16 PM

Page 14

Passage VII (Questions 3944) Short wavelength electromagnetic rays called X-rays are often used to image internal anatomical structures. Some parts of the body are nearly transparent to X-rays, while others are relatively opaque. A picture is taken of the shadow cast under X-ray illumination. The intensity of a beam of Xrays transmitted through a medium is given by I = Ioemx, where Io is the intensity incident on the medium, is the absorption coefficient of the medium, and x is the distance traveled through the medium. Like other electromagnetic waves, the frequency f and wavelength of a beam of X-rays satisfy the relations c = f and E = hf, where c is the speed of light in a vacuum, E is the energy of a photon, and h is Plancks constant. (Note: hc = 1240 nmeV.) X-rays interact with atoms primarily via collisions with electrons in low atomic energy levels. The probability of a collision with an electron in a neutral atom is proportional to the fourth power of the nuclear charge. In this type of collision, the electron is frequently ejected from the atom. A higher level electron falls down to fill the vacancy, emitting a long wavelength photon in the process and creating a new vacancy at a higher energy level. A higher level electron fills the new vacancy. This process continues until there are no electrons at higher energy levels to fall to a lower state, leaving a single vacancy. The atom is then singly-ionized. This rapid emission of photons is called a cascade. Geiger counters measure the intensity of X-rays by maintaining a high voltage difference between a wire and the metal wall of a gas chamber. Free electrons created by radiation-induced ionization are accelerated toward the wire so rapidly they ionize even more atoms in the gas. This ionization avalanche causes the counter to click.

40. X-rays are typically designated as those electromagnetic waves between about 0.01 nm and 1 nm. Which of the following energies might an X-ray photon have? A. B. C. D. 1 MeV 10 keV 100 eV 10 eV

41. The X-rays that doctors actually look at are, in fact, negatives of the shadow. Bones and connective tissue are usually light, while soft tissue is dark. This is because: A. the greater relative concentration of heavy elements found in bones and connective tissue scatters more of the incident X-rays. heavy elements are more common in bones and connective tissue resulting in greater emission of the lower energy cascade photons. heavy elements are less common in bones and connective tissue allowing more incident X-rays to show through. the light elements in soft tissue absorb the cascade photons, increasing the relative brightness of the unscattered X-rays.

B.

C.

D.

42. Compared to the incident X-rays, the cascade photons have: A. B. C. D. lower energy. shorter wavelength. higher speed. higher frequency.

39. Tin has ten naturally occurring non-radioactive isotopes, with atomic masses of 112, 114, 115, 116, 117, 118, 119, 120, 122, 124. Which of these would you expect to scatter the most X-ray radiation? A. B. C. D. The isotope with atomic mass 112 The isotope with atomic mass 124 A mixture of isotopes in the natural abundances All would scatter X-rays equally well

GO ON TO THE NEXT PAGE. 14

Full-Length Test 5

06/27/2003

12:16 PM

Page 15

43. What is the ratio of the intensity of an X-ray beam after it passes through 1 cm of plasma to the intensity of the beam after it passes through 6 cm of plasma? (Note: Plasma has an absorption coefficient of approximately 0.2 cm1.) A. B. C. D. 1/e e0.2 e e2

44. In order for the free electrons in a Geiger counter to be accelerated toward the wire: A. B. C. D. the wall of the gas chamber must be at a lower potential than the wire. the wall of the gas chamber must be at a higher potential than the wire. the wall of the gas chamber and the wire must be connected by an equipotential surface. the wire must be negatively charged.

GO ON TO THE NEXT PAGE. 15

Full-Length Test 5

06/27/2003

12:16 PM

Page 16

Passage VIII (Questions 4550) Gravimetric analysis is an analytical technique based on the measurement of weight. This technique often reveals information about the composition of a compound. Thermogravimetric analysis (TGA) monitors weight as a function of temperature. The apparatus is shown in Figure 1. The sample to be analyzed is placed in a cup that is encased in a gas-filled chamber. The nature of the gas can be varied to make the atmosphere around the sample oxidizing, reducing, inert, etc. The furnace surrounding the chamber ensures the temperature of the sample can be varied while its weight is still monitored.
Inlet Gas Microbalance Sample Cup Furnace

50

CaC2O4H2O Compound A Compound B Compound C

500 Temperature (C)

1000

Figure 2 Graph of sample weight vs. temperature 45. Which of the following accurately represents the reaction between nickel and oxygen? A. B. C. D. 2Ni(s) + 3/2O2(g) Ni2O3(s) Ni2+(aq) + O2(aq) NiO(s) Ni(s) + 1/2O2(g) NiO(s) Ni(s) + O2(g) NiO2(s)

46. How many moles of product are formed when nickel reacts with oxygen? A. B. C. D. 0.86 0.43 8.6 104 4.3 104

Exit Gas

Figure 1 TGA Apparatus A student decided to carry out two experiments using TGA. In the first experiment, nickel powder (50 mg) was placed in the sample cup and the chamber was evacuated and then filled with oxygen at a rate of 0.1 L/min. The sample was then heated to 1000 C at a rate of 10 C/min. Upon heating, the weight of the sample increased to 64 mg. The sample was then cooled to room temperature and the chamber was evacuated. During this time, the weight of the sample did not change. However, upon filling the chamber with hydrogen and heating the sample as before, the weight of the sample dropped to 50 mg. In a second experiment, 50 mg of calcium oxalate (CaC2O4H2O) was placed in the sample cup. The chamber was filled with air, and the sample was heated to 1000 C at a rate of 10 C/min. The student measured the weight of the sample at 50 C intervals, and obtained the following graph: 47. Would TGA be a suitable method to detect the occurrence of a phase change in a compound? A. B. C. D. Yes, because the phase change is accompanied by an increase in weight. Yes, because the phase change is accompanied by a decrease in weight. No, because the molecular structure of the sample does not change. No, because the phase change is not accompanied by a change in weight.

GO ON TO THE NEXT PAGE. 16

Full-Length Test 5

06/27/2003

12:16 PM

Page 17

48. Unlike nickel(II) oxide, iron(II) oxide is easily oxidized to iron(III) oxide. This is because: A. B. C. D. Fe3+ contains a half-filled d subshell, which is very stable. nickel cannot possess oxidation states higher than +2. Ni2+ contains a half-filled d subshell, which is very stable. iron(II) oxides are very unstable compounds.

50. When water is added to Compound C, a very strong base is formed. Compound C is most likely: A. B. C. D. calcium oxalate. calcium oxide. calcium carbonate. calcium hydroxide.

49. What products are formed when the nickel sample is treated with hydrogen? A. B. C. D. Nickel(II) oxide and water Nickel and water Nickel hydride and oxygen Nickel and hydrogen peroxide

GO ON TO THE NEXT PAGE. 17

Full-Length Test 5

06/27/2003

12:16 PM

Page 18

Questions 51 through 55 are NOT based on a descriptive passage.


51. Which of the following molecules is nonpolar? A. B. C. D. PCl3 SO2 CH3Cl BF3

54. A helicopter flying west at 10 mph starts to increase its altitude at a rate of 5 mph. If it passes through a region of space in which there exists an air current of 10 mph moving from north to south, how long will it take the helicopter to travel 3 miles? A. B. C. D. 7.2 minutes 12 minutes 18 minutes 20 minutes

52. Three charges of equal magnitudes but unequal signs are placed at the corners of an isosceles triangle. Charges Q and R are equidistant from charge P and are fixed so that they cannot move. What is the initial velocity vector of charge P as it is released?
D

55. The following reaction is an example of: Zn(s) + CuSO4(aq) ZnSO4(aq) + Cu(s) A. B. C. D. a single displacement reaction. a metathesis reaction. a decomposition reaction. a disproportionation reaction.

P +

A + Q

A. B. C. D.

A B C D

53. A block is attached to a vertical spring and set into oscillation. Which of the following will increase the period of the resulting oscillations? I. Increasing the mass of the block II. Increasing the amplitude of the oscillation III. Decreasing the spring constant of the spring A. B. C. D. I only I and II only I and III only II and III only

GO ON TO THE NEXT PAGE. 18

Full-Length Test 5

06/27/2003

12:17 PM

Page 19

Passage IX (Questions 5660) Natural light can be polarized upon reflection as shown in Figure 1. An unpolarized light beam in air is incident on a dielectric surface (glass) at an angle i equal to the Brewster angle. Since the light is incident at the Brewster angle, a right angle is formed between the refracted and reflected rays, and the reflected ray is linearly polarized. The electric field vector of the reflected ray is parallel to the glass surface. The relationship between the angle of incidence i and the angle of refraction t is given by Snells law: nAsini = nGsint, where nA is the index of refraction of air and nG is the index of refraction of glass. Figure 2 shows an unpolarized light source illuminating the linear polarizers P1 and P2 at normal incidence. Unpolarized light incident on a polarizer can be represented as the superposition of any two waves with electric field vectors that are equal in magnitude and perpendicular to each other. When unpolarized light is incident on a polarizer, the polarizer transmits 100% of the electric field component parallel to its transmission axis, but none of the perpendicular component. According to Malus law, the intensity of light transmitted through polarizer P2 is given by I1cos2, where I1 is the intensity of the light transmitted through P1, and is the angle between the transmission axes of the polarizers P1 and P2. (Note: cos = sin(90 ))

P1 Light Source A

P2 B

Figure 2 Unpolarized light incident on polarizers P1 and P2

56. The orientation of the transmission axis of a polarizer can be determined by looking through it into the reflected beam of Fig. 1. Which statement accurately describes what is observed as the polarizer is rotated? A. The light intensity transmitted through the polarizer is at a minimum when its transmission axis is parallel to the glass surface. The light intensity transmitted through the polarizer is at a minimum when its transmission axis is perpendicular to the glass surface. The light intensity transmitted through the polarizer is at a minimum when its transmission axis is at an angle of 45 with respect to the glass surface. The light intensity transmitted through the polarizer is independent of the polarizers orientation.

B.

C.

D.
lig ht sou rce

r
glass

57. Longitudinal waves, such as sound waves cannot be polarized because:


t

A. B. C. D.

Figure 1 Unpolarized light incident on glass at the Brewster angle

they are not reflected at the boundary between two media. they are not attenuated when they pass from one medium to another. they have no electric field component. their vibration is not perpendicular to the propagation direction.

GO ON TO THE NEXT PAGE. 19

Full-Length Test 5

06/27/2003

12:17 PM

Page 20

58. What is the intensity at point A? A. B. C. D. 0 I0/4 where I0 is the intensity of the unpolarized light incident on P1 I0/2, where I0 is the intensity of the unpolarized light incident on P1 I0, where I0 is the intensity of the unpolarized light incident on P1

60. Which of the following represents the Brewster angle for the light in Figure 1? A. B. C. D. 90 0 t 90 t

59. If the transmission axes of P1 and P2 are perpendicular to each other, what would be the effect of inserting a third linear polarizer at point A with its transmission axis at a 45 angle with respect to those of the other polarizers? A. B. C. D. The intensity of the light beam at point B would increase. The intensity of the light beam at point B would decrease. The intensity of the light beam at point B would not change, but its polarization would change. The polarizer would have no effect on the intensity or the polarization at point B.

GO ON TO THE NEXT PAGE. 20

Full-Length Test 5

06/27/2003

12:17 PM

Page 21

Passage X (Questions 6166) On a spherical planet of mass M and radius R, the acceleration due to gravity is g = GM/R2, where G is the universal gravitational constant. The magnitude of the escape velocity from the surface of the planet is vesc = 2G M/R. This is the minimum velocity needed to escape the pull of the planets gravity. Depending on the temperature, mass, and size of a planet, many particles in its atmosphere may have speeds large enough to escape into interstellar space. Even on the Earth, certain gases which are common to the Sun and outer planets have long ago leaked away into interstellar space. This is because at a given temperature, particles with greater speeds leak away more rapidly. The probability P(v) that a gas particle has speed v is shown in Figure 1, for a gas at two different temperatures. This is known as the Maxwell speed distribution. Notice that the probability goes to zero as the speed goes to infinity. As the temperature increases, high speed particles become more probable. For an ideal gas of particles in thermal equilibrium at temperature T, the root-meansquare speed per particle is 3kBT/m, where kB is a constant equal to 1.38 1023 J/K and m is the mass of a particle. Since kinetic energy is 1/2 mv2, the average translational kinetic energy per particle is 3/2 kBT. (Note: Assume that all of the gases referred to in the questions behave like ideal gases.)
T1 T1 < T2

61. The mass of Jupiter is about 363 times that of the Earth, and Jupiter has a diameter 11 times as great. Approximately, what is the ratio of the magnitude of the escape velocity on Jupiter to that on Earth? A. B. C. D. 3/11 11/3 33 11 3

62. What is the minimum kinetic energy a particle of mass m must have to escape from the surface of the Earth? A. B. C. D. GmME/RE2, where ME is the mass of the Earth and RE is the radius of the Earth GmME/RE, where ME is the mass of the Earth and RE is the radius of the earth 3/2 kBT, where T is the temperature at the surface of the Earth 1/2 mvrms2, where vrms is the root-mean-square speed of the particle

63. Consider nitrogen and oxygen gas at the same temperature. Which of the following statements is most correct? A. B. The maximum speed of the oxygen molecules is greater than that of the nitrogen molecules. The average translational kinetic energy of the oxygen molecules is equal to that of the nitrogen molecules. Each oxygen molecule has about 4/3 more translational kinetic energy than each nitrogen molecule. The friction between the oxygen and nitrogen molecules raises the temperature of the mixture.

C.

T2 P(v)

D.

Figure 1 Maxwell speed distribution of particles of a gas at temperatures T1 and T2 where T1 < T2

GO ON TO THE NEXT PAGE. 21

Full-Length Test 5

06/27/2003

12:17 PM

Page 22

64. Although the composition of the Earths atmosphere is mostly stable over human lifetimes, composition shifts do change over billions of years. Neglecting shifts due to climate and ecology, one would expect that the ratio of: A. B. C. D. hydrogen to oxygen will decrease. water vapor to ammonia will decrease. nitrogen to oxygen will increase. methane to carbon dioxide will increase.

66. The fact that the Earth is rotating about its polar axis affects the escape velocity from the surface of the planet. Taking into account the Earths rotation, the escape velocity at the North Pole is: A. B. C. D. greater than the escape velocity at the South Pole. less than the escape velocity at the South Pole. greater than the escape velocity at the equator. less than the escape velocity at the equator.

65. Assuming the Moon once had an atmosphere, the best explanation for why the Moon currently has no atmosphere is: A. B. gaseous elements never existed on the Moon. gaseous elements escaped from the Moon because the temperature on the Moon is too high for its size and mass. gaseous elements escaped from the Moon because the temperature on the Moon is too low for its size and mass. the magnetic field of the Moon is too weak.

C.

D.

GO ON TO THE NEXT PAGE. 22

Full-Length Test 5

06/27/2003

12:17 PM

Page 23

Passage XI (Questions 6772) Water that contains cations capable of forming precipitates is defined as hard. Such cations include Mg2+, Ca2+, and Fe2+; all of which are capable of forming slightly soluble carbonate salts. These carbonate salts can build-up in the water supply as boiler scalesolid deposits that can block water pipes if not removed accordingly. The concentration of carbon dioxide in water can affect the precipitation of salts such as calcium carbonate. Dissolved carbon dioxide reacts with water to form carbonic acid (H2CO3) which in turn dissociates to form a bicarbonate ion (HCO3) and a proton (Reaction 2). This bicarbonate ion can react with Ca2+ to form a precipitate (Reaction 5), or dissociate further to form carbonate (Reaction 3). The carbonate ion can also react with Ca2+ to form a precipitate (Reaction 4). CO2(g) CO2(aq) H = 19.41 kJ/mol Reaction 1 CO2(aq) + H2O(l) H2CO3(aq) Reaction 2 HCO3(aq) H+(aq) + CO32(aq) Reaction 3 Ca2+(aq) + CO32(aq) Reaction 4 Ca2+(aq) + 2HCO3(aq) CO2(g) + H2O(l) + CaCO3(s) Reaction 5 Table 1 gives relevant data for various species in Reactions 2 through 5. Ka HCO3(aq) H2CO3(aq) CO3
2(aq)

67. Based on the information in the passage, HCO3 is: A. B. C. D. a strong acid and a weak base. a weak acid and a weak base. a strong acid and a strong base. a weak acid and a strong base.

68. Assuming that H2CO3 only dissociates into HCO3 and H+, what is the pH of a 0.023 M H2CO3 solution? A. B. C. D. 1.0 2.0 4.0 8.0

69. A reservoir contains water saturated with both Mg2+ and Ca2+ ions. Which carbonate salt will constitute most of the precipitate? (Note: Ksp(MgCO3) = 1.6 106.) A. B. C. D. CaCO3, because its Ksp is smaller than that of MgCO3. MgCO3, because its Ksp is larger than that of CaCO3. There will be an equal mixture of salts. Neither MgCO3 nor CaCO3 will precipitate.

H+(aq) + HCO3(aq)

CaCO3(s)

Kb 2.33 1.79 Table 1 104 4.5 108

Ksp 109

5.61 4.3

1011 107

CaCO3(s)

GO ON TO THE NEXT PAGE. 23

Full-Length Test 5

06/27/2003

12:17 PM

Page 24

70. Which of the following could be added to a solution of H2CO3 to make an acidic buffer? A. B. C. D. H2SO4 Na2SO4 NaHCO3 NaCl

72. Which of the following factors will affect the molar solubility of CO2(g) in a reservoir? I. The pH of the water II. The temperature of the water III. The partial pressure of CO2(g) A. B. C. D. I only II only I and III only I, II and III

71. According to the information in the passage, will an increase in pH result in increased precipitation of CaCO3? A. Yes; increasing the pH shifts the equilibria of Reactions 2 and 3 in favor of carbonate ion production. Yes; increasing the pH increases the proton concentration and shifts the equilibria of Reactions 2 and 3 in favor of carbonate ion production. No; proton concentration does not affect the equilibria of Reactions 2 and 3. No; increasing the pH shifts the equilibria of Reactions 2 and 3 in favor of H2CO3 production.

B.

C. D.

GO ON TO THE NEXT PAGE. 24

Full-Length Test 5

06/27/2003

12:17 PM

Page 25

76. Which of the following will not form an ionic crystal?

Questions 73 through 77 are NOT based on a descriptive passage.


73. A 2-kg object slides down an incline with an angle of 30 to the horizontal. If the force of friction is 2-N, what is the acceleration of the object? A. B. C. D. 9.8 m/s2 8 m/s2 5 m/s2 3.9 m/s2

A. B. C. D.

MgF2 CO2 NaCl KNO3

77. What percentage of a sample of a radioactive element with a half-life of 4 days remains after 16 days? A. B. C. D. 4% 6.25% 8% 12.5%

74. What is the most stable outer electron configuration of Cr0? A. B. C. D. 4s24p4 4s23d4 4s03d6 4sl3d5

STOP. IF YOU FINISH BEFORE TIME IS CALLED, CHECK YOUR WORK. YOU MAY GO BACK TO ANY QUESTION IN THIS SECTION ONLY.

75. What is the pH of a 0.1 M aqueous NH3 solution when 1% of ammonia undergoes hydrolysis? A. B. C. D. 3 7 11 13

25

Full-Length Test 5

06/27/2003

12:17 PM

Page 26

Full-Length Test 5

06/27/2003

12:17 PM

Page 27

Verbal Reasoning
Time: 85 Minutes Questions 78137

DO NOT BEGIN THIS SECTION UNTIL YOU ARE TOLD TO DO SO.

Full-Length Test 5

06/27/2003

12:17 PM

Page 28

VERBAL REASONING
DIRECTIONS: There are nine passages in this Verbal Reasoning Section. Each passage is followed by several questions. After reading a passage, select the one best answer to each question. If you are not certain of an answer, eliminate the alternatives that you know to be incorrect and then select an answer from the remaining alternatives. Indicate your selection by blackening the corresponding oval on your answer document.
Passage I (Questions 7883) Neoconservatives increasingly blame our problems: maleducation, victimism, criminal behavior, whatever... on the 1960s, as if in those years America abruptly plunged from Apollonian civilization to Dionysian chaos. Recently, the date when the U.S., or more precisely when many people within it, began to tip off the emotional tracks was fixed as August 1968. Supposedly, our national life unraveled when protesters battled Chicago police and leftist intellectuals condoned them... Violence of all kinds, and excuses for it, was multiplying well before 1968. More to the point, the Chicago demonstratorsand many murderers, rapists, muggers and rioterswere born in the '40s and early '50s, and raised in the Eisenhower years, supposedly an era of wholesome nuclear families informed by the right values. If we accept the premise of family values advocates, that the child is father to the man, then the '60s mayhem proves that something was wrong with millions of American families in the '50s, and that, moreover, the foundations of civilized life were crumbling long before Chicago... The ideas informing the much-maligned 60s leftist intellectuals (Progressive educators, and prophets of permissiveness like Dr. Spock) were already wellentrenched... Even before World War I, many influential intellectuals had rejected the Protestant ethic of selfrestraint. American capitalisms shift to mass consumption, already established in the 1920s, undermined that ethic for the whole society... Rebellion against restraint, including self-restraint, was central to the Romantic movement. Glorification of spontaneity and the noble savage unsullied by civilization started with the Enlightenment, as did the rise of determinism, which rationalizes misconduct and denies personal responsibility... In 1897, while God was in His church, Queen Victoria was on her throne, America was on the gold standard, and all was right with the world; socialism, anarchism and pragmatism were gaining popularity; Darwinism and higher criticism were 28
40

already undermining religion; Western high culture was rotten with fin de siecle Decadence, inspired by, among others, the Marquis de Sade; and John Dewey wrote My Pedagogic Creed, launching American education on the road to ruin. But subversive intellectuals and cultural influences could not have succeeded unless they were telling people what they wanted to hear. Ultimately, the blame for our lack of guardrails lies with flawed human nature. We sinners have our choices. The buck stops with the sinner, not with the tempter. Moreover, subversives were crashing an open door. Authority flopped at defending the civilization it guarded. So did such idols of neoconservatisms pantheon as...the family. Authority not only neglected the guardrails, or actually removed them, it set a disastrous example by crashing them... Which family values dominated the much-lauded 50s? For the first time, entertainment and indulgence of children was an unofficial national priority, from Disneyland and allowances to cars given upon graduation from high school. Intensely materialistic, their own childhoods harrowed by the Depression, fatally equating indulgence with love, too many American parents were as far removed from the Romans who told their sons to return with their shields or on them as the Keystone Cops are from the Coldstream Guards. If their kids got into trouble at school or elsewhere, they typically sided with them and tried to bail them out. They put comfort over character and raised their children to fit in, be popular, make money, and have fun. During this same time, the much-bewailed illegitimacy rate had already more than tripled between 1940 and 1960, from 7.1 per thousand unmarried females of childbearing age to 21.8. Playboy appeared in 1953; presently, Elvis lewdly bucked his pelvis against his guitar to cheers from middleclass audiences; and in 1959, a federal judge ruled that Lady Chatterleys Lover wasnt obscene, and that sending it through the mail was legal, opening the door to torrents of fabricated satyriasis... Righteousness ran a poor second. GO ON TO THE NEXT PAGE.

45

10

50

15

55

20

60

25

65

30

70

35

75

Full-Length Test 5

06/27/2003

12:17 PM

Page 29

80

Todays lack of guardrails isnt the '60s fault. No great nation goes to pieces so abruptly as that. The '60s did not cause our moral collapse. They only unmasked it. But the worst aspect of slapdash neoconservatives penchant for blaming the '60s is that it understates the problem. The assault on the beliefs underpinning civilized life, and the failure to uphold them, have been going on not for 30 years but for centuries...

81. The author cites all of the following as contributors to societal moral collapse (line 78) EXCEPT: A. B. C. D. the Marquis de Sade. Darwinism. violence. lax parental discipline.

82. Which of the following statements, if true, would most weaken the authors argument? A. 78. Although not explicitly stated, the author would agree that nontraditional, free-thinking behavior began: A. in the 1950s with the proliferation of automobiles, dating, teenage pregnancy, and musicians lewd lyrics and gestures. in the late 1800s because of mass destruction of societal infrastructure from the Civil War. centuries ago with movements such as the Enlightenment and determinism. during the Great Depression, as individuals were forced to forgo concerns or proper societal behavior and concentrate on the more basic needs of food and shelter. B. The illegitimacy rate prior to 1940 was always below 0.71 percent. Only a small percentage of children raised in the 1950s achieved great financial wealth during their lifetimes. Other nations have maintained traditional morality and stemmed societal violence despite exposure to pornography, capitalistic mass consumption, and free thought. Crime rates in western society have risen in inverse proportion to mean income levels and religious devotion.

C.

B. C. D.

D.

83. By stating,...all was right with the world...(lines 3536), the author implies that: A. no dissenting philosophies were followed by great masses. although global society was at complete peace, some dissension existed. societal leaders correctly viewed western society as properly structured. conservatives were naive in believing a utopian society existed.

79

In which of the following would one most reasonably expect to find this passage? A. B. C. D. a graduate sociology thesis the editorial page of a scholarly journal a commentary in a history textbook a speech

B. C. D.

80. If spoken to the author, the statement, Our government makes no sense unless it is founded in a deeply felt religious faithand I dont care what [that faith] is would be regarded by the author as: A. B. C. D. a fundamental tenet of healthy democracy. alien to neoconservative beliefs. indicative of staggering intellectual vacuity. an essential doctrine of Progressive educators.

GO ON TO THE NEXT PAGE. 29

Full-Length Test 5

06/27/2003

12:17 PM

Page 30

Passage II (Questions 8490)


50

Some blacks had made livings as entertainers as far back as the eighteenth century, but the tradition of the black entertainer really began after the Civil War, when blacks were free to travel and set up businesses of their 5 own. They began by offering minstrel shows in imitation of the ones white entertainers had been presenting since the 1830s; when the minstrel show gave way to the variety theater in the 1880s, many black dancers, singers, and comics went along with it. By the 1890s, black entertain10 ers were an accepted part of show business. Nor was it just entertainers: By the end of the century black writers and composers were beginning to create fulldress musical comedies for Broadway theaters. These shows frequently included white acts and , perhaps more surprisingly, played to mixed audiences, with the blacks seated either upstairs or in half the orchestra sets. Indeed, by 1910, there had grown up a vogue for black shows written and directed by men like Rosamond Johnson, his brother James Weldon Johnson, Will Marion Cook, and Will Vodery, two of whom would act as tutors to Duke Ellington... Black show business had its ups and downs; according to Thomas Laurence Riis, who has made a careful study of the early black musical theater, these black shows at times had trouble getting bookings in the best houses and were forced into second-tier ones, which some refused to play. Then, in 1921, a musical called Shuffle Along, cooked up by the black entrepreneurs Flournoy E. Miller and Aubrey Lyles, with music by Eubie Blake and Noble Sissle, became a smash hit. The immensely popular song, Memories Of You, came from that show. Josephine Baker was in the chorus, and it skyrocketed Florence Mills, on whose death [Duke] Ellington would compose a tribute, to fame. The show started a renewed interest in black entertainment that would become a boom by the end of the decade. According to J.A. Jackson, a black columnist who reported regularly on black entertainment for Billboard, the show began the renaissance of the Negro in musical comedy. As early as 1923, Jackson would write, Things are picking up very perceptibly for the colored group in this industry due to the increase in the numbers of pictures being made in the territory adjacent to New York. Another factor in the matter is that directors are desirous of having naturalness and have just about eliminated the madeup, white actor in Negro characterizations. With the success of Shuffle Along and the dozens of black shows that followed over the decade, cabaret managers began booking the black acts in white locations. By

1923 Florence Mills was the headliner at the Plantation Room on Broadway; in 1924 the jazz dancer Johnny Hudgins was at the Winter Garden; and in 1923, a group of young musicians from Washington would make it to Hollywood.

84. Based on the information in the passage, which of the following statements is NOT true? A. B. C. D. The musical Shuffle Along recharged Americas interest in black musical theater. Duke Ellington was taught by the most talented musicians of the early 1920s. Black artists often accepted bookings in secondtier venues. Black artists gradually gained acceptance as entertainers in the nineteenth century.

15

20

85. Which of the following statements, if true, would most strongly support the authors assessment regarding the impact of Shuffle Along? A. B. C. D. After Shuffle Along, many black artists were booked into mixed-audience clubs. The songs written for Shuffle Along are still popular today. Black artists experienced a renaissance in the 1920s. Both black and white audiences enjoyed black entertainers in Shuffle Along.

25

30

35

86. According to the passage, in which decade did black artists achieve the greatest degree of integration in theater? A. B. C. D. The 1880s The 1920s The 1890s The 1830s

40

45

GO ON TO THE NEXT PAGE. 30

Full-Length Test 5

06/27/2003

12:17 PM

Page 31

87. The author suggests that black artists resurgent popularity in the 1920s was primarily due to: A. B. C. D. the freedom to travel and operate business after the Civil War. audiences unpopular attitudes toward white actors in heavy makeup. white audiences desire to see black performers. an increase in film-making around New York at the same time.

90. The passage discusses all of these critical points in the history of black entertainment EXCEPT: A. B. C. D. the creation of musical comedies by black artists. the development of the black entertainer in response to the Civil War. the immediate consequences of the success of Shuffle Along. the role of the black entertainer in the eighteenth century.

88. The author mentions Josephine Baker (lines 3132) in order to: A. underscore Shuffle Alongs importance to the development and dispersal of black theater and black performers. support his opinion that the success of Shuffle Along lies in its talent. draw attention to a black artist made famous by Shuffle Along. provide evidence of the popularity of black artists to white audiences.

B. C. D.

89. The author makes all of the following assertions EXCEPT: A. B. C. by the end of the 1920s, the black musical theater found renewed, widespread acceptance. black entertainers first succeeded by imitating white entertainers. full-dress musical comedies were the sole province of white artists until the decade after the Civil War. the black musical renaissance was not limited to black audiences.

D.

GO ON TO THE NEXT PAGE. 31

Full-Length Test 5

06/27/2003

12:17 PM

Page 32

Passage III (Questions 9196) Major wars often provide the punctuation marks of history, primarily because they force drastic realignments in the relationships among states. To this rule the First World War was no exception. Long before the fighting ceased in 5 November 1918, it was evident that the map of Europe must be redrawn and that reallocation of colonies, creation of a new international organization, and change in the economic balance must considerably affect the rest of the world as well. The First World War heralded the end of 10 European dominance, as the true victors in this predominantly European war were America and Japan: two nonEuropean powers. The European victors were bled white and suffered a pyrrhic victory from which none of them ever really recovered. While this fact was not evident at 15 the wars end, it was clear that the forthcoming settlement must far exceed in geographic scope and complexity those other periodic realignments of the power balance, the 1648 Treaty of Westphalia and the 1815 Final Act of Vienna, to which it is often compared. Nobody doubted the magni20 tude of the task ahead but nobody was properly prepared to undertake it. As often happens, the sudden collapse of the enemy took the victors by surprise. Germany had been expected to hold out until mid-1919, and in the autumn of 1918, 25 Allied energy was more concentrated upon winning the war than upon planning the peace. True, some planning was in progress, but not always in the most effectual quarters. In the final year of the war, the smaller Allied states pursued their limited, specific aims with energy, but 30 achieved only cautious and qualified commitments. Exile organizations representing ethnic groups within the Central powers did the same with similar results. They recognized that the ultimate court of appeal would consist of Britain, France, and America, but these three, who had the 35 task of planning for much of the world, were also responsible for winning the war. Not surprisingly, that came first.

91. Based on the information presented in the passage, the author would most likely agree with which of the following statements? A. B. Britain, France, and the United States were prepared for the rapid demise of Germany. European states never really recovered from the realignment of power that followed the 1815 Final Act of Vienna. Europes dominance of world politics ended with World War I. Creation of a new international organization preoccupied the minds of the victorious powers before the war was won.

C. D.

92. In the context of the passage, the phrase pyrrhic victory (line 13) means: A. B. C. D. a total defeat. an overwhelming victory. a costly triumph. a disastrous truce.

93. If true, which of the following arguments would most effectively weaken the authors argument? A. B. C. The European powers eventually recovered from World War I. Without intervention, the economic balance of Europe stabilized rapidly after the war ended. The 1648 Treaty of Westphalia was even more complex than the agreement that ended World War I. World War I caused no drastic change in Europes central importance to global politics.

D.

GO ON TO THE NEXT PAGE. 32

Full-Length Test 5

06/27/2003

12:17 PM

Page 33

94. The passage suggests that the United States, Britain and France: A. B. C. D. were largely responsible for the direction and aims of the peace process. could have ended the war much sooner. doubted the peace process would take very long. paid close attention to the claims of smaller Allied nations in the late stages of the conflict.

96. Based on the information in the passage, which of the following is NOT true? A. B. C. D. Ethnic factions within the Central Powers won only cautious promises from the Allied nations. Germanys early collapse in 1918 found the Allied nations somewhat unprepared for peace. The Allied nations knew the peace process would be a monumental task. The reallocation of colonial possessions was not considered before the peace process began.

95. The second paragraph suggests that: A. B. C. D. the early plans for peace were mostly made by ineffectual leaders. Allied forces were intently focused on winning the war in the fall of 1918. Germanys war effort was expected to last into the 1920s. wars run a natural, predictable course.

GO ON TO THE NEXT PAGE. 33

Full-Length Test 5

06/27/2003

12:17 PM

Page 34

Passage IV (Questions 97103) In Prisoners Of Mens Dreams, Suzanne Gordon argues that American feminism has lost sight of its original goal of transforming the world into a kinder, gentler place. Gordon deplores the sort of feminism that has triumphed 5 instead: cold, ruthless, equal-opportunity feminism, which aims for womens entrance into the masculine public world and their achievement by male standards of excellence. The heart of the book consists of excerpts from a hundred interviews with career women, who do a lot of complaining about fatigue and disillusion. At the end, Gordon calls for a National Care Agenda that would make caregiving rather than competition the ultimate American value. Suzanne Gordon is obviously an intelligent, sympathetic, and well-meaning person, but Prisoners Of Mens Dreams is a good example of the kind of sentimental, unlearned effusion that has become a staple of contemporary feminism and that most men rightly ignore. Like so many American feminists, Gordon is completely out of her depth as a social analyst. Awkward, unintegrated quotes from Adam Smith and Woodrow Wilson are waved around to disguise her lack of familiarity with economics, history, and political science. We are presented with the usual three-handkerchief tear-jerker scenario about Big, Bad, Ugly Americathat corrupt, empty, greedy society which all those wonderful, warm, benevolent people around the world look at with disgust. This point of view is the essence of chic these days among know-nothing feminists and the preening pseudoleftists who crowd our university faculties. Well, let me tell you: As a child of Italian immigrants, I happen to think that America is the most open, dynamic, creative nation on Gods green earth. As a scholar, I also know that it is capitalist America that produced the modern independent woman. Never in history have women had more freedom of choice in regard to dress, behavior, career, and sexual orientation. Gordons insistence on defining women as nurturant and compassionate drove me up the wall. My entire rebellion as a child in the Fifties was against this unctuous, preachy stuff coming from teachers, nuns, and Girl Scout leaders. Gordons caregiving transformative feminism is just as repressive and reactionary as the patriarchy it claims to attack. Minerva save us from the cloying syrup of coer-

cive compassion! What feminism does not need, it seems to me, is an endless recycling of Doris Day Fifties cliches about noble womanhood.

10

97. The author of the passage would most likely claim that someone who did NOT agree with her view of feminism was: A. B. C. D. independent. sentimental. rebellious. matriarchal.

15

98. According to the author, which of the following statements is NOT true? A. B. C. D. Works like Prisoners of Mens Dreams are routinely ignored. Gordons National Care Agenda is well-meaning. Gordon has experience as a social analyst. Gordons feminist rhetoric equates womanhood with nurture and care.

20

25

99. The main argument of the passage is that: A. B. C. D. Gordon is an intelligent woman. Gordon finds America repressive, corrupt and greedy. Gordons argument reinforces outdated feminist stereotypes. Doris Day is not a good feminist model.

30

35

40

45

GO ON TO THE NEXT PAGE. 34

Full-Length Test 5

06/27/2003

12:17 PM

Page 35

100. In the passage, transformative feminism (line 43) is used to mean: A. B. C. D. a political agenda with caregiving as its guiding principle. the process of reinventing power structures to provide equal opportunities for women. encouraging people to adopt womanly virtues as cultural standards. a goal of redefining feminine roles to include nurturing and compassion.

103. The authors opinion of Gordons view of American values can best be described as: A. B. C. D. disinterested. tolerant. uninformed. dismissive.

101. Which of these assumptions is NOT implicit in the authors view? A. B. C. D. Stereotypes concerning female attributes are incorrect. Equal-opportunity feminism is fundamentally ideal. Gordons rationale gives credence to Fiftiesstyle cliches of feminine roles. American values and mores are considerably more liberal than those of other nations.

102. The authors most strenuous objection to Gordons thesis is that: A. B. C. D. it lacks scholarly rigor. it offers political solutions for moral questions. it depicts modern feminism as cold and ruthless. the modern independent woman is free to choose her dress, career, and behavior.

GO ON TO THE NEXT PAGE. 35

Full-Length Test 5

06/27/2003

12:17 PM

Page 36

Passage V (Questions 104111) Welles had directed himself many times on the stage, but somehow the very permanence of film, the possibility of displaying one version of a role was somewhat formidable to him. On the stage he wouldand did change his approach to a part almost from night to night to better hone it into shape and to sense what an audience, a particular audience, needed or wanted from any given performance. Once the curtain went up, the performances success or failure was the actors own: he had no director to say Cut! if he was bungling his lines or losing his audience. This fluidity was impossible when working with the terminal, packaged quality of a finished motion picture. The camera never laughed, nor did it fall asleep. It was impossible for it to gauge ones performance. Chaplin had been successful in directing himself on film, but there were two important differences between what he had accomplished and what Welles was attempting to do: Until that time, Chaplin had worked exclusively with silent film, and all his films were comedies. Although it is true that drama did not contain the demands of pacing found in comedy, the dimension of spoken dialogue could add enormous problems for someone appraising himself. Welles, however, was choreographing Citizen Kane with the unhesitating measure and continual rhythm of an uptempo musical comedy or, as someone has since suggested, a grand opera. So that he could see how he would ultimately move amid and through a particular set and piece of business, he always had on hand a double, not in the traditional acting sense of someone who looked like him, but an actor who was as nearly exact in his height, weight, and general bodily structure as possible. Sometimes he had William Alland, although smaller and thinner, walk through his part for him. When the young and eager Kane bounced out of the horsedrawn carriage and into the building of the New York Inquirer, for instance, it was Welles double who first went through the motion without benefit of camera, those that Welles the actor would ultimately make on the screen. That was the easy part. More difficult was going through the gestures himself, first in rehearsal, and then for the initial shootings. How is one to know that a slight trembling of the lips, or the blink of the eye, or an awkward movement of the shoulder, will not be recorded by the camera? He found it difficult hitting his own chalk marks, that predetermined place where he was to stand so that [the camera] could better work [its] magic. On the stage he could accidentally, or by choice, be slightly more stage left or right, but with film, although he still had to maintain rhythm and endurance with body movements, it was imperative that he hit these marks accurately and with absolute precision every time. 36

55

It took only a few sessions of directing himself and others, followed by a careful examination of the rushes, to prove how intrinsically different stage and screen acting really were. The lesson of the whole history of the art of acting in films began to wash over him: the fact that some of Eisensteins best actors had never seen the inside of a theater; that some of the greatest stage performersBernhardt and Duse and Beerbohm Tree among themwere disappointments on the screen.

10

104. The main idea of the passage is that: A. Welles approached the challenge of directing himself on film in exactly the same manner as he had done on stage. Welles improvised techniques, such as using a double, to free himself to examine his staging techniques. Welles found only through experience that film directing is quite different than stage directing. the great actors of the stageBernhardt, Duse, and Beerbohm Treewere unable to translate their ability to film.

15

B.

20

C. D.

25

105. The author mentions Chaplin (lines 1519) primarily to: A. B. C. D. compare Welles difficulties to those of another great actor who had directed himself on film. contrast Welles pacing of Citizen Kane to the pacing of a comedy film. underscore the fundamental change that spoken dialogue had brought into film-making. introduce the notion that Welles choreography was similar to that of a grand opera.

30

35

40

106. If true, which of the following would most weaken the authors argument? A. B. C. D. Welles quickly adapted his film-acting skills to the stage. Stage acting uses the same timing and placement techniques as screen acting. Chaplin had worked in both silent films and talkies with great success. The success of a stage production depends on the interaction of the cast. GO ON TO THE NEXT PAGE.

45

50

Full-Length Test 5

06/27/2003

12:17 PM

Page 37

107. The term rushes (line 52) means: A. B. C. D. a careful sketch made from staging ideas. spots marked on the stage to cue actors positions. unedited film footage. changes made to the script by the director.

110. Which of the following is asserted with no supporting evidence? A. B. C. Some of the greatest stage actors were disappointments on the screen. Welles used another actor to assume his parts during the planning stages of his films. As a stage director, Welles had more flexibility than he did in film work. William Alland was not an exact physical double for Welles.

108. It can be inferred from the passage that Welles: A. B. found that drama films are more difficult to pace than comedies. made his directors job much more difficult by pacing Citizen Kane like an up-tempo musical comedy. directed himself more successfully than did Chaplin. found it easier to gauge his performances on film than on stage.

D.

111. According to the passage, which of the following is true about stage acting technique? A. B. C. It denies the actor a certain freedom of expression. It promotes a fixed interpretation of primary characters. It requires less physically exacting technique and timing than does film. It translates easily to film.

C. D.

109. According to the passage, the author would describe Welles stage acting technique as all of the following EXCEPT: A. B. C. D. fluid. changeable. formulaic. responsive.

D.

GO ON TO THE NEXT PAGE. 37

Full-Length Test 5

06/27/2003

12:17 PM

Page 38

Passage VI (Questions 112118)


50

10

A behavior as complex as sleepwith its highly differentiated component non-REM and REM phasesis unlikely to be dedicated to any one particular function, yet folk wisdom has tended to collaborate with scientific reductionism in supposing that one, and only one, function is served by sleep. The universal favorite candidate for this function is rest. This is probably the carryover of the naive notion that in sleep our brains are at a low and monotonous level of activity. Yet in spite of years of research, science has not definitely established even one function for sleep. It is at least logically appropriate to assume, as our mothers did, that sleep is necessary for health since we subjectively experience sleep as restful and restorative. So overwhelmingly clear is the sense of restoration following a good night of sleep that no one in his or her right mind would abandon the rest theory despite the deafening silence of physiology on this question. In this sense, the subjective experience of sleep should more powerfully motivate us to seek physiological or behavioral explanations, just as the experience of dreaming should have motivated scientists to look for evidence of brain activation during sleep. But the rest theory is likely to require a more specialized answer in the case of the kind of sleep we humans share with our fellow mammals. The reason is that rest does not require sleep. Inactivity alone should suffice to provide us with rest. And many organisms already spend a good deal of their wake-state time at rest. The rest function may well be further elaborated in higher animals with complex brains so as specifically to restore efficiency to such crucial wake-state functions as attention and memory. Such rest is likely to be associated with more active processes than the simple passive one the rest theory would imply. For example, while the brainmind is freed of the task of monitoring and remembering new information in sleep, it can review and reorganize its own already acquired data. It is in this sense that most of the new hypotheses arising from modern sleep research distinguish themselves from Freuds contributions and those of his contemporaries and predecessors. The reason it has been difficult to establish convincing functional hypotheses is, as usual, methodological. The most obvious experimental approach to functional questions is to deprive people of sleep and then observe any behavioral deficits. But anyone who has undergone a night of self-imposed sleep deprivation will know that this approach is not only painful but difficult. So many things need to be done in order to keep oneself awake that it is 38

virtually impossible to control for nonspecific and unintentional effects of the deprivation procedure itself. These nonspecific procedural factors may cause the deficits in performance. And despite all efforts to stay awake, we doze off anyway! Preferable to sleep deprivation would be some measure of brain function (or behavioral capability, or psychological process) that could be tested around the clock and be found to (1) deteriorate as the wake period is prolonged and (2) recover dramatically following an epoch of sleep. This positive functional model, in which sleep would be shown to reverse a process that declines progressively in waking, has never been successfully applied to any functional question. Even if one were successful in completely preventing sleep and effectively controlling the inadvertent side effects of deprivation, one would then have to move to a molecular or cellular neurobiological approach to understand the positive effects of such deprivation. Researchers have convincingly demonstrated that sleep loss can be fatal. Sleep-deprived rats fail to regulate their energy and literally consume themselves metabolically. Now the question is: How are such effects mediated? This example shows clearly that sleep deprivation per se is only an instrumental tool; it is not an analytic probe. It may help us to ask the right question but it can never, by itself, give us the answers we seek.

55

60

15

65

20

70

25

75

30

112. The passage suggests that sleep researchers inability to establish convincing functional hypotheses indicates that: A. B. C. D. the hypotheses of Freud and his predecessors must still be considered potentially valid. the theory that the purpose of sleep is to allow the organism to rest must be eliminated. researchers have experienced difficulty devising an appropriate experimental approach. sleep may not in fact have any particular function aside from rest.

35

40

45

GO ON TO THE NEXT PAGE.

Full-Length Test 5

06/27/2003

12:17 PM

Page 39

113. Which of the following, if true, would most weaken the authors argument against the theory that the sole function of sleep is simply to provide passive rest? A. B. Folk wisdom and science seldom agree in their explanations of natural phenomena. Selective deprivation of sleep during each of its component phases has varying behavioral effects. Some species of mammals spend over 50% of their awake time at rest. Measures of attention and memory remain stable regardless of time spent in sleep.

116. Suppose that a medical resident, who must stay awake during work-shifts of 36 hours duration, is given a simple memory test at the beginning and the end of one such shift. Her score is found to be significantly lower at the end of the shift. Which of the following responses is most consistent with the passage? A. The experiment establishes that sleep deprivation affects memory function but doesnt provide information as to how this occurs on a molecular level. The reason memory capacity decreased is that the resident was required to retain a lot of new information and wasnt able to review and reorganize it during sleep. Because many factors could have affected the residents memory capacity over the course of the shift, it cannot be concluded that the memory deficit was the result of sleep deprivation. The residents memory should have been tested during the work-shift as well as before and after.

C. D.

B.

114. On the basis of the authors comments in the second paragraph about our subjective experience of sleep, it is reasonable to conclude that the author believes that: A. subjective experience can stand alone as sufficient evidence to prove a theory about bodily functions correct. dreaming subserves some sort of physiological restorative function. a lack of scientific evidence for a theory does not automatically invalidate it. subjective experience can always be relied upon as an accurate source of information.

C.

D.

B. C. D.

117. Which of the following would be the best example of the successful application of the authors positive functional model (line 59) to a sleep-function question? A. A subjects mood is evaluated before and after sleep deprivation and is found to have been improved by deprivation. The overall level of brain electrical activity in a subject is measured over a 24-hour period and is found to oscillate regularly. The concentration of a neurotransmitter in a subjects brain is found to decrease over the course of the day but is restored after sleep. A subjects reading speed is tested after a night of sleep and is found to be nearly identical to the previous days rate.

B. 115. Although no supporting evidence is provided, the author claims that: A. B. C. D. the function of sleep cannot be determined simply by observing the effects of sleep deprivation. brain activity is at a low and monotonous level during sleep. many organisms spend a good deal of their wake-state time at rest. sleep deprivation can be fatal.

C.

D.

118. The author asks the question How are such effects mediated? (line 71) in order to show that: A. B. C. D. knowing which question is the right one to ask is crucial. the dearth of knowledge about sleep at a molecular level is staggering. scientists cannot fully investigate sleep using sleep deprivation. productive scientific research always leads to further questions.

GO ON TO THE NEXT PAGE. 39

Full-Length Test 5

06/27/2003

12:17 PM

Page 40

Passage VII (Questions 119124) ...Can the university, with its special trust of protecting free speech, be hampered by the restrictions of civility? What kind of a frill is etiquette, anyway, for those in the noble pursuit of truth?
5 45

Jurisprudence uses etiquette in courtroom procedure, not only to restrict speech but to impose standards of dress, comportment and forms of addressmatters over which universities have long since abandoned authority. Legislators and diplomats also know the value of keeping speech within the bounds of civility. The parliamentary etiquette book, Roberts Rules of Order, proscribes disorderly words and forbids speakers to arraign the motives of a member during strongly worded debate. It is not the man, but the measure, that is the subject of debate, decrees its section on Decorum. The rougher the conflict, the more manners are needed. Only when insults, harassment, disrespect and obscenity are banned can people engage in truly substantive argument. Of course it is also a personal insult to call someone a racist or a sexist. Incivility is no more acceptable in defense than in attack. Rebuttal, however, is a staple of open debate. Members of the university community should always have the opportunity to attack ideasbut not to attack people. The university should be obliged to provide a forum for anyone who wants to argue for or against an idea, provided the argument is made in good faith and a polite manner. The standard of academic etiquette must be required not only in the classroom and lecture hall but wherever the community of scholars gathersresidence halls, dining commons, recreational facilities. Invective, whether spoken or conveyed through posters or graffiti, in the classroom or in the community, is detrimental to rational debate, to which universities are dedicated.

10

These questions are raised whenever a loose-tongued student turns publicly nasty. When Brown University recently expelled such a student, many argued that all restrictions of free speech are intolerable in the university. Browns President, Vartan Gregorian, agreed with that premise and neatly reclassified the offensive speech as behavior. But the premise is wrong. The special trust of a university is not to foster unlimited speech. It is to foster unlimited inquiry. And totally free speech inhibits rather than enhances the free exchange of ideas. The law cannot restrict such speech without violating our constitutional rights. But etiquette, the extra-legal regulative system that seeks to avert conflict before it becomes serious enough to call in the law, can and does. You may have a legal right to call your mother an idiot, or somebody elses mother a tramp, but you wont if you know whats good for you. Nor could you convince many people that the controversy that such remarks are likely to provoke will lead to advances in knowledge. The university needs to enforce rules banning speech that interferes with the free exchange of ideas. It must protect the discussion of offensive topics but not the use of offensive manners. It must enable people freely to attack ideas but not one another. Education is impossible without the order that prevents intimidation and mayhem. When children first enter school, they must be taught to sit still, refrain from taunting their classmates, show respect for their teacher and wait their turn to talk, or they will never be able to learn. To those who find it horrifying that the university should allow a lesser degree of free speech than the law permits, it might be pointed out that the law itself restricts free speech in its pursuit of juridical truth. Try saying some of the things in a courtroom that the law will protect your right to say in a barroom.

50

55

15

20

65

25

70

30

35

40

GO ON TO THE NEXT PAGE. 40

Full-Length Test 5

06/27/2003

12:17 PM

Page 41

119. According to the essay, for what can you be expelled from a university? I. Standards of comportment. II. Offensive behavior. III. Offensive speech. A. B. C. D. II only I and III only III only I, II and III

122. The authors define the university as: A. B. C. D. a forum dedicated to rational debate. a forum that must protect the discussion of offensive topics. an institution of higher learning. a forum for invective.

123. According to the passage, in what location does the law protect your right to free speech? A. B. C. D. In parliament In residence halls In a bathroom In a barroom

120. According to the passage, what is the strongest argument for the use of etiquette versus the first amendment? A. B. C. D. The use in our courtrooms The use in parliament Racist remarks not leading to advances in knowledge Incivility being no more acceptable in defense than attack

124. In defense of restrictions on offensive speech, the Fourteenth Amendment is often cited. It guarantees equal protection of the laws to all, including those who are targets of offensive speech. The authors main response to this would probably be: A. all restrictions of free speech are intolerable in the university. the law cannot restrict such speech without violating our constitutional rights. the university must protect the discussion of offensive topics but not offensive ideas. reclassify the offensive speech as offensive behavior.

121. To what are the authors referring when they say, But the premise is wrong (line 12)? A. B. C. D. Offensive behavior The expulsion of a student on the basis of offensive speech Intolerable restrictions of free speech in the university Restrictions of civility hampering the university

B. C. D.

GO ON TO THE NEXT PAGE. 41

Full-Length Test 5

06/27/2003

12:17 PM

Page 42

Passage VIII (Questions 125132)


50

10

...[post-World-War-II Director of Policy Planning George F.] Kennans strategy had been to try to bring about changes, over time, in the Soviet concept of international relations: to convince Russian leaders that their interests could be better served by learning to live with a diverse world than by trying to remake it in their own image. Kennan had rejected both war and appeasement as means of accomplishing this; it could only be done, he thought, through a long-term process of what might be called behavior modificationresponding positively to whatever conciliatory initiatives emanated from the Kremlin, while firmly countering those that were not... Kennan took the position that modifying Soviet behavior required both positive and negative reinforcement: it was as important to reward the Kremlin for conciliatory gestures as it was to resist aggressive ones. This meant being prepared to engage in such negotiations that would produce mutually acceptable results. The [Truman] administration conveyed the appearance of being willing to discuss outstanding issues with Moscow, but Kennan regarded several of its major actions between 1948 and 1950...[among them] the formation of the North Atlantic Treaty Organization (NATO)...as certain to reinforce Soviet feelings of suspicion and insecurity, and hence, to narrow opportunities for negotiations... The initiatives for the North Atlantic Treaty came from the West Europeans themselves, and reflected the uneasiness they felt over the disparity in military power on the European continent: the Russians had, at that time, thirty divisions in Eastern and Central Europe alone; comparable combined U.S., British, and French forces came to less than ten divisions. Intelligence estimates credited the Russians with the ability to sweep to the English Channel and the Pyrenees in a matter of weeks By September 1948 the Western Union countries (Great Britain, France, and the Benelux countries), together with the United States and Canada, had agreed on the outlines of a treaty providing that an attack on any one of them or on any other nations that might be included within the terms of the treaty would be regarded as an attack upon all. Because of a trip to the Far East and a subsequent period of hospitalization, Kennan had not been involved in the initial discussions regarding European security early in 1948, but upon his return he quickly made clear his reservations about the course the administration had chosen to follow. These boiled down to three points: (1) That the Europeans had mistaken what was essentially a political

55

60

threat for a military one, and that they risked, as a result, a general preoccupation with military affairs, to the detriment of economic recovery; (2) that outside the immediate North Atlantic area, which embraces a real community of defense interest firmly rooted in geography and tradition, any alliance extended to only some countries would render the rest all the more vulnerable...; (3) that an alliance made up of [Western European] nations would amount to a final militarization of the present dividing-line through Europe, and that no alteration, or obliteration, of that line could take place without having an accentuated military significance. Such a development might be unavoidable, but our present policy is still directed...toward the eventual peaceful withdrawal of both the United States and the U.S.S.R. from the heart of Europe... These were not isolated concerns. There was worry in Washington that emphasis on rearmament would delay recovery; indeed, one of the conditions attached to the administrations military assistance program for Western Europe was that economic revival would continue to have first priority. The question of how to include some countries without appearing to write off others also caused a great deal of agonizing: In the end, the administration stretched the concept of North Atlantic to encompass Italy, but refused to extend it to Greece, Turkey, Iran or to form a comparable pact with the non-communist countries of the Western Pacific. There was less concern about Kennans third pointthat an alliance might freeze the existing division of Europesimply because most observers already regarded that division, by mid-1948, as an accomplished fact... Despite its reservations, the administration went on to conclude a North Atlantic Treaty and initiate a program of military assistance to its members. Kennan came to see, regretfully, that [because of military insecurity of Europeans] there were few alternatives... [The decision on NATO]...was not intended to preclude eventual negotiations with the Russians, but it did seek to defer them until requisite levels of strength had been reached. In so doing, however, it left little room for efforts to alter the Soviet concept of international relations through positive as well as negative reinforcement.

15

65

20

70

25

75

80

30

85

35

40

90

45

GO ON TO THE NEXT PAGE. 42

Full-Length Test 5

06/27/2003

12:17 PM

Page 43

125. The passage suggests that Kennans behavior modification (line 10) approach to changing the Soviet concept of international relations was: A. B. C. D. unlikely to be successful if the Kremlin always made conciliatory gestures. moderate in comparison with the approaches he decided to reject. a logical outgrowth of his extensive background in behavioral psychology. an extension of American strategy during the Second World War.

128. Kennan assumed which of the following in making his first point (lines 4752) about NATO? A. B. C. D. The formation of the military alliance would spur economic growth. The presence of the thirty Soviet divisions did not mean they were going to attack. The economic recovery in Europe had been progressing slowly. Its always a mistake to make military affairs a higher priority than economic affairs.

126. Based on the passage, Kennans attitude about heightening Soviet insecurity and suspicion through the formation of NATO was one of: A. B. C. D. belief that it was appropriate negative reinforcement of the Soviet military presence in Europe. hope that it would prompt the Soviets to make more conciliatory gestures. disapproval because it would make modifying Soviet international behavior much more difficult. belief that negotiations with the Soviets would be easier on a more even playing field.

129. Kennans reaction to the administrations refusal to extend NATO membership to Greece, Turkey or Iran was most likely one of: A. understanding, because these countries did not have the same geographic defense interests as the Europeans. approval of the fact that the concept of North Atlantic was not over extended. disappointment that those countries could not now be employed in anti-Soviet strategy. trepidation that these countries were now more open to potential enemy aggression.

B. C. D.

127. The passage suggests that the impetus for the formation of NATO was: A. B. C. information that a Russian attack on Western Europe was impending. the understanding that no nation could withstand a Russian attack without assistance. the desire to aid the Western European economic recovery as well as to guarantee military assistance. the fear that the Soviets would try to capitalize on their military advantage.

130. Which of the following explains why the Truman administration was not worried about Kennans objection that NATO would amount to a final militarization of the present dividing-line through Europe? A. They believed that it would be possible to alter the line through negotiations of peaceful withdrawal. They wanted to maintain a strong American military presence in Europe. They felt it was too late to prevent the solidification of the dividing line. Neither of Kennans other two objections to NATO had given them cause for concern.

B. C. D.

D.

GO ON TO THE NEXT PAGE. 43

Full-Length Test 5

06/27/2003

12:17 PM

Page 44

131. The passage suggests that, with regard to the reservations expressed by Kennan about NATO, the administration was: A. B. C. D. often in agreement but ultimately undeterred. unresponsive to his proposals for improvement. able to counter each of his criticisms. forced to carefully re-examine its objectives.

132. Which of the following conclusions would be most in accord with a theme of the passage? A. B. Military alliances invariably have drawbacks that render them ineffective. Behavioral modification is the only way to change a governments concept of international relations. Coherent international strategy can flounder because of the military situation. Negotiations should be conducted between two powers once military equilibrium has been established.

C. D.

GO ON TO THE NEXT PAGE. 44

Full-Length Test 5

06/27/2003

12:17 PM

Page 45

GO ON TO THE NEXT PAGE. 45

Full-Length Test 5

06/27/2003

12:17 PM

Page 46

Passage IX (Questions 133137) The new field of molecular anthropology has provided important new data on primate evolution, but many paleontologists have been slow to accept them. They argue that fossils are the only direct evidence and that molecules are not relevant to the study of extinct species. With regard to human origins, fossil bones and teeth may appear to conflict with molecular data, but, as we have seen with Ramapithecus, the fossil evidence is often fragmentary and subject to more than one interpretation. The two approaches are quite different: Paleontologists dig up bones from the earth, study their shapes, and compare them to other living and extinct species in order to establish their place in an evolutionary phylogeny. Molecular anthropologists study the proteins of living species and deduce how long ago two species diverged from a common ancestor... Proteins are made up of various combinations of the basic twenty amino acids, arranged in definite sequences. A given protein may include hundreds to thousands of amino acids. The proteins of closely related species, such as horse and donkey or dog and fox, are nearly identical; whereas species that diverged more than 100 [million years ago], such as shrew and opossum, have many sequence differences. These differences can be measured precisely, and their number is approximately proportional to the divergence time. Such molecular clocks are particularly valuable for evolutionary study because results can be, and have been, replicated in numerous laboratories, whereas the analysis of fossil bones and teeth is somewhat subjective, and agreement between researchers may be difficult to achieve. Paleontological phylogenies have indicated a long, independent evolution for the five living hominid groups. They have suggested that Pliopithecus was an early gibbon, that different species of Dryopithecus were ancestral to gorilla, chimpanzee, and orangutan, and that Ramapithecus was ancestral to the human line. In contrast, the molecular phylogenies show a relatively recent divergence for all the apes, with the Asian apes splitting off earlier than the rest. This rules out the possibility that Pliopithecus is an ancestral gibbon or Dryopithecus or Proconsul are ancestral apes. The three-way divergence between human, chimpanzee, and gorilla about 5 [million years ago] does not preclude Australopithecus as an early representative of the human family. So, aside from the equivocal fossil evidence itself, the molecular findings make it even less likely that Ramapithecus could 46
50

have been a human ancestor. To consider Ramapithecus as hominid, one would have to assume that primate proteins have evolved at half the rate of shark, fish, frog, snake, kangaroo, mouse, and elephant proteins. Some have argued that proteins evolve more slowly in animals with more time between generationsa presumption refuted by the evidence that mouse and elephant proteins have evolved at the same rate, as have loris and human proteins. This external check on the statistical constancy of the molecular clock is further supported by internal evidence: numerous proteins with different rates of change (cytochrome c, albumin, transferrin, hemoglobin, and histones) indicate similar divergence times. That is like timing the same event with an hour hand, minute hand, and second hand and finding out that the times come out the same. The statistical constancy of the molecular clock is not an assumption but an observation based on an enormous amount of data. From the biochemical evidence, then, there was but one ancestor prior to the ape-human split, not three different ones, for humans and the African apes. The molecular information had made it clear that we cannot look at human origins as an isolated phenomenon, but must consider the event as part of the radiation that included the African apes. If Ramapithecus has a place in human ancestry, it will have a place in chimpanzee and gorilla ancestry as well.

55

10

60

15

65

20

70

25

30

133. The author would agree with which of the following statements regarding the origins of the five living hominid groups? I. Pliopithecus was an early gibbon. II. Dryopithecus and Proconsul are ancestral apes. III. Australopithecus is a human ancestor. A. B. C. D. I only I and II only III only I, II and III

35

40

45

GO ON TO THE NEXT PAGE.

Full-Length Test 5

06/27/2003

12:17 PM

Page 47

134. Based on the information presented in the passage, what would the author believe is most significant to anthropologists regarding the findings of molecular phylogenies? A. B. C. It is less likely that Ramapithecus could have been a human ancestor. Fossil evidence is fragmentary and subject to more than one interpretation. Ape and human origins prior to the ape-human split are much more closely related than was previously thought. The invention of the molecular clock was made possible.

137. The author claims, but offers no supporting evidence, that: A. B. C. D. the analysis of fossil bones and teeth is subjective. Ramapithecus is ancestral to the gorilla. the molecular clock has statistical constancy. we cannot look at human origins as an isolated phenomenon.

D.

STOP. IF YOU FINISH BEFORE TIME IS CALLED, CHECK YOUR WORK. YOU MAY GO BACK TO ANY QUESTION IN THIS SECTION ONLY.

135. Which of the following statements would be most relevant to the first paragraph of the passage? A. Almost 200 years ago, French paleontologist Cuvier boasted that he could reconstruct an animal from its teeth alone. But paleontologists have learned the hard way that teeth do not tell the whole story. Ramapithecus, first found in India, was named in the 1930s after the Hindu prince Rama, hence Ramas ape. The famous nineteenth-century anatomist Thomas Henry Huxley, in his 1863 book, Evidence As to Mans Place in Nature, argued on anatomical grounds that humans were most similar to the African apes, a conclusion confirmed 100 years later by biochemistry. A comparison of developmental stages among vertebrates led Ernst Haeckel (18341919) to propose his famous principle ontogeny recapitulates phylogeny.

B.

C.

D.

136. Which of the following would most strengthen the claim that molecular anthropology has practical value? A. B. C. D. Evidence that ape and human proteins have evolved at the same rate The finding by paleontologists that the Asian apes did not split off earlier than the rest Evidence that Australopithecus was an early human The finding of a complete intact fossil specimen of Ramapithecus with characteristics that are as much apelike as human

47

Full-Length Test 5

06/27/2003

12:17 PM

Page 48

Full-Length Test 5

06/27/2003

12:17 PM

Page 49

Writing Sample
Time: 60 Minutes 2 Prompts, Separately Timed: 30 Minutes

DO NOT BEGIN THIS SECTION UNTIL YOU ARE TOLD TO DO SO.

Full-Length Test 5

06/27/2003

12:17 PM

Page 50

WRITING SAMPLE
DIRECTIONS: This is a test of your writing skills. The test contains two parts. You will have 30 minutes to complete each part. Your responses to the prompts given in this Writing Sample will be written in the ANSWER DOCUMENT. Your response to Part 1 must be written only on the answer sheets marked 1, and your response to Part 2 must be written only on the answer sheets marked 2. You may work only on Part 1 during the first 30 minutes of the test and only on Part 2 during the second 30 minutes. If you finish writing on Part 1 before the time is up, you may review your work on that part, but do not begin writing on Part 2. If you finish writing on Part 2 before the time is up, you may review your work only on part 2. Use your time efficiently. Before you begin writing a response, read the assignment carefully and make sure you understand exactly what you are being asked to do. You may use the space below each writing assignment to make notes in planning your responses. Because this is a test of your writing skills, your response to each part should be an essay composed of complete sentences and paragraphs, as well organized and clearly written as you can make it in the alloted time. You may make corrections or additions neatly between the lines of your responses, but do not write in the margins of the answer booklet. There are six pages in your answer booklet to write your responses, three pages for each part of the test. You are not required to use all of the pages, but to be sure that you have enough space for each essay, do not skip lines. Essays that are illegible cannot be scored.

GO ON TO THE NEXT PAGE. 50

Full-Length Test 5

06/27/2003

12:17 PM

Page 51

Part 1
Consider this statement: In the recording of history, it is impossible to be objective. Write a unified essay in which you perform the following tasks. Explain what you think the above statement means. Describe a specific situation in which it might be possible to be objective in the recording of history. Discuss what you think determines when objectivity in the recording of history is possible and when it is not.

GO ON TO THE NEXT PAGE. 51

Full-Length Test 5

06/27/2003

12:17 PM

Page 52

Part 2
Consider this statement: The best politician is the one most removed from politics. Write a unified essay in which you perform the following tasks. Explain what you think the above statement means. Describe a specific situation in which the best politician might not be the one most removed from politics. Discuss what you think determines whether or not the best politician is the one most removed from politics.

GO ON TO THE NEXT PAGE. 52

Full-Length Test 5

06/27/2003

12:17 PM

Page 53

Biological Sciences
Time: 100 Minutes Questions 138214

DO NOT BEGIN THIS SECTION UNTIL YOU ARE TOLD TO DO SO.

Full-Length Test 5

06/27/2003

12:17 PM

Page 54

BIOLOGICAL SCIENCES
DIRECTIONS: Most of the questions in the Biological Sciences test are organized into groups, with a descriptive passage preceding each group of questions. Study the passage, then select the single best answer to each question in the group. Some of the questions are not based on a descriptive passage; you must also select the best answer to these questions. If you are unsure of the best answer, eliminate the choices that you know are incorrect, then select an answer from the choices that remain. Indicate your selection by blackening the corresponding circle on your answer sheet. A periodic table is provided below for your use with the questions.

PERIODIC TABLE OF THE ELEMENTS


1 H 1.0 3 Li 6.9 11 Na 23.0 19 K 39.1 37 Rb 85.5 55 Cs 132.9 87 Fr (223) 4 Be 9.0 12 Mg 24.3 20 Ca 40.1 38 Sr 87.6 56 Ba 137.3 88 Ra 226.0 21 Sc 45.0 39 Y 88.9 57 La * 138.9 89 Ac 227.0 22 Ti 47.9 40 Zr 91.2 72 Hf 178.5 104 Rf (261) 58 Ce 140.1 90 Th 232.0 23 V 50.9 41 Nb 92.9 73 Ta 180.9 105 Ha (262) 59 Pr 140.9 91 Pa (231) 24 Cr 52.0 42 Mo 95.9 74 W 183.9 106 Unh (263) 60 Nd 144.2 92 U 238.0 25 Mn 54.9 43 Tc (98) 75 Re 186.2 107 Uns (262) 61 Pm (145) 93 Np (237) 26 Fe 55.8 44 Ru 101.1 76 Os 190.2 108 Uno (265) 62 Sm 150.4 94 Pu (244) 27 Co 58.9 45 Rh 102.9 77 Ir 192.2 109 Une (267) 63 Eu 152.0 95 Am (243) 64 G d 157.3 96 Cm (247) 65 T b 158.9 97 Bk (247) 66 Dy 162.5 98 Cf (251) 67 Ho 164.9 99 Es (252) 68 Er 167.3 100 Fm (257) 69 Tm 168.9 101 Md (258) 70 Y b 173.0 102 No (259) 71 Lu 175.0 103 Lr (260) 28 Ni 58.7 46 Pd 106.4 78 Pt 195.1 29 Cu 63.5 47 Ag 107.9 79 Au 197.0 30 Zn 65.4 48 Cd 112.4 80 Hg 200.6 5 B 10.8 13 Al 27.0 31 Ga 69.7 49 In 114.8 81 Tl 204.4 6 C 12.0 14 Si 28.1 32 Ge 72.6 50 Sn 118.7 82 Pb 207.2 7 N 14.0 15 P 31.0 33 As 74.9 51 Sb 121.8 83 Bi 209.0 8 O 16.0 16 S 32.1 34 Se 79.0 52 Te 127.6 84 Po (209) 9 F 19.0 17 Cl 35.5 35 Br 79.9 53 I 126.9 85 At (210) 2 He 4.0 10 Ne 20.2 18 Ar 39.9 36 K r 83.8 54 Xe 131.3 86 Rn (222)

GO ON TO THE NEXT PAGE. 54

Full-Length Test 5

06/27/2003

12:17 PM

Page 55

Passage I (Questions 138143) Retinoblastoma (RB) is a cancer of the retinal cells that develops in children between birth and the age of 4. RB is caused by mutations in the RB gene, located on chromosome 13. These mutations may be inherited from a parent (familial RB) or may arise spontaneously (sporadic RB). The wild-type RB gene is a tumor suppressor genea gene that normally restrains cell growth. The wild-type RB gene product, the protein p105-RB, inhibits cell progression from the G1 phase to the S phase of the cell cycle. Since RB develops when both copies of the wild-type gene are lost or inactivated, the RB gene is sometimes referred to as a recessive cancer gene. In familial RB, a child is born with one inherited copy of the mutant RB gene, and one normal copy. The RB is triggered by a somatic mutation in a retinal cell that results in the loss of the normal RB gene. This acquired homozygosity causes the retinal cell to give rise to a tumor. Sporadic RB is the result of two somatic mutations in a single retinal cell after birth. Loss of both normal copies of the RB gene leads to the same tumor growth associated with familial RB. However, whereas familial RB patients are usually blinded at an early age by multiple RB tumors, sporadic RB can develop at any age and typically produces only one tumor. Types of mutations that inactivate the RB gene include large deletions, small deletions affecting promoter sites, splicing errors that remove exons, and point mutations.

139. What is the probability that a man with familial RB will pass the mutant Rb gene on to a daughter? A. B. C. D. 25% 50% 75% 100%

140. According to the passage, p105-RB inhibits cell progression from the G1 phase to the S phase of the cell cycle. This implies that p105-RB most likely: A. B. C. D. inhibits microtubule formation, thus inhibiting cell movement. inhibits DNA synthesis, thus inhibiting mitosis. stimulates protein synthesis, thus promoting cell growth. stimulates mutagenesis by binding to the RB gene, thus promoting RB development.

141. The protein products of tumor suppressor genes may function in all of the following ways EXCEPT: A. B. C. D. as an inhibitor of cell growth. as an activator of programmed cell death. as a regulator of cellular differentiation. as an activator of cellular proliferation.

138. If R = the allele for the wild-type RB gene and r = the allele for the mutant RB gene, then what is the genotype of an RB cancer cell in a child with familial RB and in a child with sporadic RB, respectively? A. B. C. D. rr and rr Rr and RR Rr and Rr rr and RR

GO ON TO THE NEXT PAGE. 55

Full-Length Test 5

06/27/2003

12:17 PM

Page 56

142. Some retinal cells that are homozygous mutant at the RB gene loci still produce the p105-RB protein. Which of the following best explains how this occurs? A. B. C. D. There is incomplete penetrance of the mutant RB gene. The p105-RB produced by these retinal cells is not functional. The mutation in one of the RB genes is not in a transcription promoter site. The retinal cells are phenotypically normal.

143. Some RB cancers are initiated by infection with SV40, a DNA virus. However, after infection by SV40, both copies of the RB gene in a retinal cell are still normal and functional. Thus, the best explanation for the cancerous phenotype is that: A. B. C. D. p105-RB cannot be synthesized after viral infection. SV40 inserts into an exon in the RB gene. the paternal germline is more susceptible to viral infection. an SV40 protein complexes with p105-RB, thereby inactivating it.

GO ON TO THE NEXT PAGE. 56

Full-Length Test 5

06/27/2003

12:17 PM

Page 57

Passage II (Questions 144149) In an attempt to develop a vaccine for pneumonia, Fred Griffith performed a series of experiments in 1928 using mice and two strains of the pneumococcus bacteria: a virulent encapsulated strain and a nonvirulent unencapsulated strain. The encapsulated strain was called the smooth strain because the colonies looked smooth on a culture plate due to their polysaccharide capsules, whereas the unencapsulated strain was denoted as the rough strain due to the irregularity of its surface. Four different groups of mice were injected with different combinations of the bacterial strains. The experimental results are shown below in Table 1. Table 1 Results of Griffiths experiment Mice Injected Bacteria Result 1 2 3 4 live smooth live rough heat-killed smooth heat-killed smooth + live rough died survived survived died

144. Which of the following experimental observations would most contradict Griffiths interpretation? A. B. Live smooth-strain bacteria transformed roughstrain bacteria into smooth-strain bacteria. The polysaccharide capsule from the smoothstrain bacteria did not transform rough-strain bacteria. DNA from smooth-strain bacteria transformed rough-strain bacteria into smooth-strain bacteria. The transforming agent in the Group 4 mice was comprised mainly of amino acids.

C. D.

145. Which of the following most accurately describes why the heat-killed smooth strain did NOT kill the Group 3 mice? A. B. C. D. The bacteria lacked polysaccharide capsules. The bacterias DNA was denatured. The bacteria could not cause infection. The bacteria could not undergo cell division.

146. The Group 4 mice died because: A. B. C. D. the rough strain transferred its messenger RNA to the heat-killed smooth strain. the heat-killed smooth strain transferred its transfer RNA to the rough strain. the heat-killed smooth strain transferred its polysaccharide capsules to the rough strain. the heat-killed smooth strain transferred its DNA to the rough strain.

Autopsies performed on Group 4 mice revealed blood samples filled with live smooth-strain bacteria. Some component had been transferred from the heat-killed smooth-strain bacteria to the live rough-strain bacteria, transforming them into the virulent smooth-strain bacteria. This process later became known as transformation. Griffith believed that protein from the dead smooth-strain bacteria was the active transforming agent.

GO ON TO THE NEXT PAGE. 57

Full-Length Test 5

06/27/2003

12:17 PM

Page 58

147. A colony of smooth strain bacteria is grown on a culture containing an experimental drug that cleaves nucleic acid base sequences wherever adenine is paired with uracil. Which of the following processes will be directly affected? I. Transcription II. Translation III. Transformation A. B. C. D. I only I and II only I and III only I, II, and III

149. If Griffith had injected a fifth group of mice with a combination of heat-killed rough strain and heatkilled smooth strain, would the mice have died? A. B. C. D. No, because the heat-killed rough strain cannot infect mice. No, because the heat-killed smooth strain cannot transform mice cells. Yes, because the heat-killed rough strain is transformed into the smooth strain. Yes, because the mice do not have acquired immunity to pneumococcus bacteria.

148. Which of the following best describes the shape of the pneumococcus bacteria? A. B. C. D. Rod-shaped Spiral Spherical Elliptical

GO ON TO THE NEXT PAGE. 58

Full-Length Test 5

06/27/2003

12:17 PM

Page 59

Passage III (Questions 150154) Carboxylic acids (general formula = RCOOH) are very useful organic compounds, since they can be converted into a wide range of derivatives. In addition, they can be synthesized from a number of precursors. The most common reactions are summarized in Figure 1.
O RCH2OH I 1) KMnO4OH 2) H+ O R C VIII R'OH Cl SOCl2 R C II H R 1) Ag2O, OH 2) H+ O R C V OH O R C VI O R' O Na+ R X III Mg/Ether MgX IV

3 2

10

5 (ppm)

Figure 2 1H NMR Spectrum of Compound D

1) CO2 2) H+

R'OH, H+/ O R C VII

150. What is the correct IUPAC name for Compound D? A. B. C. D. Cyclohexanecarboxylic acid 2-Methylbenzoic acid Methyl benzoate 3-Methoxybenzaldehyde

(Note: R = alkyl or H; Compound V = carboxylic acid.) Figure 1 Common reactions of carboxylic acid and its derivatives To investigate these reactions, a student carried out the following procedure: Compound A (C7H6O) was reacted with silver oxide (Ag2O) to form Compound B (C7H6O2). Compound B was found to be insoluble in water; however, upon the addition of sodium hydroxide, Compound B dissolved in the aqueous layer. A mixture of Compound B, Compound C (CH4O), and mineral acid was then heated to reflux, resulting in the formation of Compound D (C8H8O2). The 1H NMR spectrum of Compound D is shown in Figure 2. 151. Which of the following is the correct structure of Compound B?
A. H3C B. C OH C. C OH D. O OH O CH3 C CH2 O CH2 C OH O

GO ON TO THE NEXT PAGE. 59

Full-Length Test 5

06/27/2003

12:17 PM

Page 60

152. Why is an acid catalyst used in the conversion of Compound V to Compound VII? A. The catalyst donates proton to the carbonyl oxygen which makes the carbonyl carbon more susceptible to nucleophilic attack. The catalyst donates a proton to the hydroxyl group which makes the carbonyl carbon more susceptible to nucleophilic attack. The catalyst donates hydroxide ions which attack the carbonyl carbon. The catalyst accepts hydroxide ions which drive the reaction to completion.

154. Would you expect Compound V to have a higher boiling point than Compound I? (Assume R = H) A. No, because Compound I will undergo intramolecular hydrogen bonding to a greater degree. No, because Compound I will undergo intermolecular hydrogen bonding to a greater degree. Yes, because Compound V will undergo intramolecular hydrogen bonding to a greater degree. Yes, because Compound V will undergo intermolecular hydrogen bonding to a greater degree.

B.

B.

C. D.

C.

D.

153. Which of the following processes will NOT result in the formation of butanoic acid? A. B. C. D. The reaction of butanol with potassium permanganate followed by acidification. The reaction of butylmagnesium chloride with carbon dioxide followed by acidification. The reaction of sodium butanoate with aqueous mineral acid. The reaction of propylmagnesium chloride with carbon dioxide followed by acidification.

GO ON TO THE NEXT PAGE. 60

Full-Length Test 5

06/27/2003

12:17 PM

Page 61

Passage IV (Questions 155159) Multiple sclerosis (MS) is a demyelinating disease afflicting oligodendrocytes, the myelin-producing cells of the central nervous system (CNS). MS is characterized by areas of demyelination, known as plaques, in both gray and white brain matter. These plaques are characterized by centers with marked reduction in oligodendrocyte density and borders that contain high concentrations of lymphocytes. The demyelinated axons passing through the plaques are largely preserved. Two theories have been advanced to explain the onset and progression of the demyelination associated with MS. Hypothesis I Oligodendrocytes are damaged, rendering them incapable of manufacturing and maintaining myelin sheaths. Scavenger cells (e.g., macrophages) take up myelin released from the damaged regions and present it to circulating T cells and B cells as a novel antigen. Once the immune cells recognize myelin as a foreign substance, they begin destroying oligodendrocytes in other regions of the CNS. Hypothesis 2 Lymphocytes found in the tissue surrounding the brains blood vessels initiate myelin degeneration through the release of lymphokines or immunoglobulins. This release may be initiated locally or as a result of an unrelated process elsewhere in the body, such as an organ fighting a viral infection. Since the myelin loss is generated by external mediators, the outer layers of myelin are destroyed first, leaving the oligodendrocytes vulnerable to attack by other cells scavenging for cellular debris.

156. In order to determine whether Hypothesis 1 is the dominant mechanism for the initiation and progression of MS, a scientist performs a series of studies. If it is shown that the immune system is the primary system involved, it can be concluded that: A. B. C. D. Hypothesis 1 is true and Hypothesis 2 is false. Hypothesis 2 is true and Hypothesis 1 is false. both Hypothesis 1 and Hypothesis 2 are false. neither Hypothesis 1 nor Hypothesis 2 can be eliminated.

157. Factors that would help explain findings of reduced nervous system function in MS patients would include all of the following EXCEPT: A. B. C. D. loss of insulating properties of myelin. decreased ability of damaged nerve axons to regenerate. diminished axon conduction velocity. diffuse central nervous system demyelination.

158. Based on information in the passage, which of the following new developments would most WEAKEN Hypothesis 2? A. B. The interface between the axon and the myelin sheath is the site of initial damage. The majority of MS patients contracted the measles virus 15 to 20 years before the onset of MS symptoms. T cells and B cells release lymphocyte stimulating substances when macrophages present them with myelin as a novel antigen. Lymphocytes can be induced to release lymphokines by chemical stimulation.

C.

D.

159. Assuming a patient has MS plaques limited to the brain, which of the following functions is most likely NOT affected? 155. A pathologist determined at autopsy that an MS patient had several large plaques in the cerebellum. When the patient was alive, all of the following functions were most likely intact EXCEPT: A. B. C. D. visual image processing. speech. fine motor coordination respiration. A. B. C. D. Eye muscle control Reasoning ability Sense of touch in arms Knee jerk reflex

GO ON TO THE NEXT PAGE. 61

Full-Length Test 5

06/27/2003

12:18 PM

Page 62

Questions 160 through 164 are NOT based on a descriptive passage.


160. The following molecules are examples of:

162. The reactivity towards SN1 reactions decreases from left to right in the following compounds:
Br Br Br

CH3

CH3

CH3

CH3

II

III

A. B. C. D.

cis/trans isomers. enantiomers. diastereomers. meso compounds.

Which of the following best accounts for this observation? A. B. The steric hindrance in the compounds increases from left to right. The reactivity of the compounds decreases from left to right. The ability of the compounds to form a planar carbocation decreases from left to right. The ability of the compounds to undergo backside attack by the nucleophile decreases from left to right.

161. Which of the following does NOT explain how it is possible for a fetus to obtain enough oxygen when blood entering the fetus has a PO of only 20 mmHg?
2

C. D.

A. B. C. D.

Fetal hemoglobin has a greater affinity for oxygen than does maternal hemoglobin. At low PO fetal hemoglobin can carry 2030% 2 more oxygen than can maternal hemoglobin. The concentration of hemoglobin in fetal blood is 50% higher than that of maternal blood. Fetal lungs are not functional prior to birth.

163. Compared to an E. coli living within the human intestinal tract, a paramecium would produce how much ATP from the degradation of one molecule of glucose? A. B. C. D. the same 1/18 as much 18 times as much 36 times as much

164. Due to the numerous similarities between modern mitochondria and bacteria, it is believed that presentday eukaryotic cells are the evolutionary product of a unique relationship between primitive aerobic prokaryotes and primitive anaerobic eukaryotes. This theory is referred to as: A. B. C. D. the chemiosmotic hypothesis. the endosymbiotic hypothesis. the coevolutionary hypothesis. the endocytotic hypothesis.

GO ON TO THE NEXT PAGE. 62

Full-Length Test 5

06/27/2003

12:18 PM

Page 63

Passage V (Questions 165169) Cortisol is a glucocorticoid steroid hormone produced by the adrenal cortex. The primary function of cortisol is to stimulate the formation of glucose from the bodys stores of protein and fat during periods of stress. Cortisol synthesis and secretion are tightly controlled in a feedback loop with the hypothalamus and the pituitary gland. Neurons in the hypothalamus release a peptide called corticotropin-releasing hormone (CRH), which causes the pituitary gland to release adrenocorticotropic hormone (ACTH). ACTH circulates in the bloodstream and stimulates the adrenals to produce cortisol. A high plasma cortisol concentration, in turn, acts on the hypothalamus to decrease CRH and on the pituitary to suppress ACTH. Cushings syndrome is defined as an excess concentration of cortisol in the bloodstream. Symptoms include hypertension, truncal obesity, diabetes, reproductive abnormalities, acne, and muscle weakness. Cushings syndrome is most commonly the result of overproduction of ACTH by the pituitary. In this case, the pituitary responds to negative feedback only at abnormally high levels of cortisol. Cushings syndrome can also be caused by adrenal tumors, by ectopic ACTH-producing tumors (such as oat cell carcinomas of the lungs), and by the administration of excessive amounts of exogenous steroids.

167. Cortisol decreases a cells utilization of glucose by inhibiting glucose transport into the cell. As a result of this, a patient with Cushings syndrome will have a(n): A. B. C. D. increased sensitivity to insulin. decreased sensitivity to insulin. decreased recovery from low plasma glucose levels. decreased plasma glucose levels.

168. Which of the following accounts for the masculine secondary sex characteristics sometimes seen in women with Cushings syndrome? A. B. C. D. Cortisol inhibits estrogen secretion. Increased ACTH stimulates adrenal androgen production. ACTH stimulates the testes to produce testosterone. ACTH converts cortisol to testosterone.

169. Dexamethasone, a synthetic glucocorticoid with greater suppressive activity than cortisol, is used as a diagnostic tool to determine the cause of Cushings syndrome in patients. If a patient is given a high dose of dexamethasone and cortisol concentration is observed to decrease, the syndrome is most likely the result of: A. B. C. D. pituitary-dependent oversecretion of ACTH. insufficient secretion of CRH. adrenal-dependent oversecretion of cortisol. over-ingestion of exogenous steroids.

165. The muscle weakness exhibited by patients with Cushings syndrome is most likely due to the: A. B. C. D. abnormally high concentration of glucose in the blood. decreased ability to respond to stressful situations. fluctuations in CRH and ACTH concentrations. increased protein catabolism.

166. Which of the following would most likely be observed in a patient with Cushings syndrome caused by an adrenal tumor? A. B. C. D. Low CRH, low ACTH, high cortisol Low CRH, high ACTH, high cortisol High CRH, high ACTH, high cortisol High CRH, low ACTH, high cortisol

GO ON TO THE NEXT PAGE. 63

Full-Length Test 5

06/27/2003

12:18 PM

Page 64

Passage VI (Questions 170174) Two of the most common carbonyl reducing agents in use today are lithium aluminum hydride (LiAlH4) and sodium borohydride (NaBH4). These reducing agents transfer a hydride ion from the reducing agent to a positively-polarized carbon. The resulting alkoxide ion then picks up a proton to form the product (Figure 1).
R H3M H R Aldehyde or Ketone C O O R C R Alkoxide ion H+ R (Note: M = B or A) H C OH CH O CH

O CHCH 1. LiAlH4' THF 2. H+, H2O Cinnamaldehyde CH CHCH2OH

Cinnamyl Alcohol

Reaction 1

O CHCH 1. NaBH4' MeOH 2. H+, H2O Cinnamaldehyde CH CHCH2OH

R Alcohol

Figure 1 Mechanism for the reduction of carbonyl compounds Although both reagents are believed to react through the same mechanism, lithium aluminum hydride is less selective than sodium borohydride (i.e., lithium aluminum hydride will indiscriminately reduce any electrophilicallypolarized carbon, whereas sodium borohydride will only reduce certain electrophilically-polarized carbons). In the reduction of conjugated carbon-carbon bonds, however, the opposite trend occurs: Lithium aluminum hydride is the more selective reducing agent. This trend is evident in the conversion of cinnamaldehyde to cinnamyl alcohol (Reaction 1 and Reaction 2). Another difference between LiAlH4 and NaBH4 is in their handling requirements. Lithium aluminum hydride reacts violently with water, so it must be dissolved in an ethereal solvent such as THF. On the other hand, since sodium borohydride is less reactive than lithium aluminum hydride, it can be dissolved in an aqueous solvent.

+
Cinnamyl Alcohol

CH2CH2CH2OH 3-Phenyl-1-Propanol

Reaction 2

GO ON TO THE NEXT PAGE. 64

Full-Length Test 5

06/27/2003

12:18 PM

Page 65

170. What is the major product of the reaction shown below?


OC2H5 O 1. LiAlH4' 2. H+, H2O

172. A chemist wished to carry out the conversion of 9fluorenone to 9-hydroxyfluorene, shown below.
O OH

9-Fluorenone

9-Hydroxyfluorene

Note =

Which of the following reagents could be used to carry out this conversion?
C.

A.

OC2H5 O

OH O

I. LiAlH4 II. Cl2/FeCl3 III. NaBH4 A. B. C. D. I only II only I and III only I, II and III

B.

OC2H5

D.

OC2H5

HO H

173. Which of the following compounds will give the highest yield of saturated product upon reaction with sodium borohydride?
A. Ph C Ph B. CHO C H D. CHO CHO C. O

171. When 1-deuteropropanal (CH3CH2CDO) is reacted with LiAlH4 followed by the addition of water, 1deutero-1-propanol (CH3CH3CHDOH) is formed. The product is most likely: A. B. C. D. an R enantiomer. an S enantiomer. achiral. a racemic mixture.

174. Which of the following reagents could NOT be used to convert cis-2-butenal to 1-butanol? A. B. C. D. NaBH4 H2/Pd H2/Ni H+/H2O

GO ON TO THE NEXT PAGE. 65

Full-Length Test 5

06/27/2003

12:18 PM

Page 66

Passage VII (Questions 175181) Human bone marrow contains cells called pluripotent hemopoietic stem cells. These cells are capable of differentiating into red blood cells, white blood cells, or platelets depending on the presence of specific growth and differentiation inducers. Production of these inducers is in turn controlled by variables outside the bone marrow. The primary factor controlling the production of red blood cells (RBCs) is the glycoprotein hormone erythropoietin. Erythropoietin is produced primarily by the kidney and is released in response to a deficiency in oxygen delivery to the tissues. Anemia is defined as a deficiency in the number of RBCs. Anemia can result from one of two primary mechanisms: insufficient RBC production (e.g., iron deficiency) or accelerated RBC loss (e.g., hemorrhaging). In both types of anemia, the concentration of hemoglobin and the hematocrit (the percentage of blood constituted by RBCs, normally 4045%) are below normal. One way to differentiate whether an anemia results from underproduction or overdestruction of RBCs is to take a reticulocyte count. Reticulocytes are immature RBCs. There will be an increase in the number of reticulocytes in the blood of patients suffering from increased RBC loss.

176. Aplastic anemia occurs when the bone marrow is not functioning. A bone marrow sample will show a hypocellular marrow. Which of the following would you expect to see in a patient with aplastic anemia? A. B. C. D. An increased number of circulating reticulocytes Iron deficiency An increased number of platelets and white blood cells, but a decreased number of RBCs A decreased number of platelets, white blood cells, and RBCs

177. In sickle cell anemia, the presence of abnormal hemoglobin makes these cells very susceptible to breakage and destruction upon exposure to oxygen. Which of the following would you expect in patients with sickle cell anemia? I. A decreased hematocrit II. An increased reticulocyte count III. White blood cell abnormalities A. B. C. D. I only I and II only II and III only I, II and III

178. In an anemic patient, you would expect to see all of the following EXCEPT: A. B. C. D. increased concentration of erythropoietin. decreased blood viscosity. increased oxygen-carrying capacity. decreased hemoglobin concentration.

175. Which of the following experimental findings most supports the conclusion that erythropoietin production is stimulated by decreased oxygen delivery and not simply decreased RBC number? A. B. C. D. Erythropoietin is secreted in response to moving to a high altitude. Erythropoietin is secreted in patients with iron deficiency anemia. Erythropoietin is secreted in response to hemorrhaging. Erythropoietin is secreted in patients with anemia due to a Vitamin B12 deficiency.

GO ON TO THE NEXT PAGE. 66

Full-Length Test 5

06/27/2003

12:18 PM

Page 67

179. Hereditary spherocytosis is an autosomal dominant disease resulting from an abnormality in the RBCs cytoskeleton. What is the probability that a child born to a man with hereditary spherocytosis and a normal woman will develop the disease? (Note: the childs paternal grandmother was unaffected.) A. B. C. D. 0% 25% 50% 100%

181. All of the following are true of RBCs EXCEPT: A. B. C. D. They cannot synthesize ATP. They are incapable of cell division. They do not have mitochondria. They are derived from the same original type of cells as white blood cells.

180. Where would you expect to find erythopoietin receptors? A. B. C. D. Inside RBCs On the cell membranes of RBCs Inside stem cells of the bone marrow On the cell membranes of stem cells in the bone marrow

GO ON TO THE NEXT PAGE. 67

Full-Length Test 5

06/27/2003

12:18 PM

Page 68

Passage VIII (Questions 182186) Receptors for the psychoactive drug delta-9-tetrahydrocannabinol (THC) have been found in the human brain and white blood cells, and more recently, in rat spleen and lymph nodes. Similar receptors have also been found in certain marine organisms, leading some researchers to suggest that THC is part of an ancient and widespread cellular signaling system. After confirming gross binding of THC to neuron and white blood cell (WBC) receptor systems in whole cell cultures, a researcher wants to compare the receptor sites themselves. He uses PCR technology to determine the DNA sequence for both the WBC and neuronal THC receptors. The researcher inserts the genetic codes for these receptors into cells grown in the lab. Three sets of cell cultures are prepared: one with the neuronal receptor, one with the WBC receptor, and a control. After 2 days of growth, all of the cultures are exposed to THC to determine binding affinity. The results are shown below. Table 1 THC binding affinity to specific receptor sites WBC receptors THC binding affinity 0.44 0.45 0.39 Neuronal receptors 1.01 0.99 1.00 Control 0.03 0.07 0.02

182. If Theory B is correct, which of the following would best account for why THC binds with less affinity to the WBC receptor than to the neuronal receptor? A. B. C. D. Competitive inhibition at the WBC receptor site Noncompetitive inhibition at the WBC receptor site WBC receptors and neuronal receptors differ in their attraction for THC WBCs have fewer membrane proteins

183. If Theory A is correct, differences in receptor affinity could best be explained by which of the following? A. B. C. There are more neurons than lymphocytes in the body. Neurons are larger than lymphocytes. Second messenger cascade reactions amplify the effect of THC binding in neurons to a much greater degree than in WBCs. The cannabinol molecule fits into the neuronal THC receptor site more easily than into the WBC THC receptor site.

D.

184. THC is found to induce synthesis of a neuropolypeptide called ASF (Appetite Stimulating Factor). In order to increase serum ASF levels, THCs site of action within the cell would most likely be: A. B. C. D. the nucleus. mitochondria. lysosomes. cilia.

Two theories are proposed to explain this difference in receptor binding. Theory A Neuronal THC receptors and WBC THC receptors have different affinities for THC. Theory B The WBC cell membrane has a membrane protein that is in proximity to the THC receptor and interferes with THC binding.

GO ON TO THE NEXT PAGE. 68

Full-Length Test 5

06/27/2003

12:18 PM

Page 69

185. Which of the following findings would favor Theory A over Theory B? A. B. C. T4 cells, which are a type of WBC, bind THC with greater affinity than all other WBCs. Radioactively labeled THC concentrates more than 2:1 in neurons versus WBCs. There are significant structural differences between the neuronal THC receptor and the WBC THC receptor. THC has enhanced potency in severely WBCdepleted AIDS patients.

186. If Theory A is correct, which of the following can most likely be concluded? A. The DNA sequence for the WBC THC receptor is different than the DNA sequence for the neuronal THC receptor. The DNA sequence for the WBC THC receptor is identical to the DNA sequence for the neuronal THC receptor. A cell poison that inhibits ATP production would not affect the experimental results. The control cell cultures had a greater concentration of WBC THC receptors than neuronal THC receptors.

B.

C. D.

D.

GO ON TO THE NEXT PAGE. 69

Full-Length Test 5

06/27/2003

12:18 PM

Page 70

189. What is the major product of the following reaction?

Questions 187 through 191 are NOT based on a descriptive passage.


CH3

CH3 C CH CH3 H+ heat

187. Which of the following animals most likely has the LEAST dense bones? A. B. C. D. Iguana Squirrel Whale Sparrow
A. CH3

CH3 OH

CH3 C CH3 CH CH2

188. Oral contraceptives contain a combination of synthetic estrogens and progestins. By which of the following methods do oral contraceptives suppress ovulation? A. The presence of high levels of estrogen and progesterone promote secretion of gonadotropinreleasing hormones, thereby inhibiting luteinizing hormone and follicle stimulating hormone. The presence of high levels of estrogen and progesterone inhibit secretion of gonadotropinreleasing hormones, thereby inhibiting luteinizing hormone and follicle stimulating hormone. The presence of high levels of estrogen and progesterone induce menstruation to occur. Estrogen and progesterone stimulate the production of human chorionic gonadotropin (HCG), which prevents ovulation from occurring.

B.

CH3 C CH3 C

CH3

CH3

C. CH3

CH3 C C CH3

B.

CH3 O D. CH2 CH3 C CH OH CH3

C. D.

GO ON TO THE NEXT PAGE. 70

Full-Length Test 5

06/27/2003

12:18 PM

Page 71

190. The reaction below is an example of:


CH2OH HO HO O

CH2O2CCH3 O

191. The role played by tropomyosin in skeletal muscle contraction is most similar to that of: A. B. C. D. an inducer. an immunoglobulin. a repressor. a catalyst.

(CH3CO)2O CH3CO2 Pyridine CH3CO2 OH OH

CH3CO2

O2CCH3

A. B. C. D.

electrophilic addition. nucleophilic substitution. hemiacetal formation. esterification.

GO ON TO THE NEXT PAGE. 71

Full-Length Test 5

06/27/2003

12:18 PM

Page 72

Passage IX (Questions 192198) The human body responds to starvation by using different substances for its primary metabolic fuel. During the first day of starvation, low blood glucose inhibits insulin secretion and stimulates glucagon secretion. Consequently, glycolysis is inhibited and glycogen utilization and glucose synthesis are promoted. In adipose cells, triacylglycerols are hydrolyzed, leading to the release and oxidation of fatty acids. In muscle, protein is degraded to provide some of the precursors for glucose synthesis. Glycerol derived from triacylglycerol cleavage is another raw material used for glucose synthesis. The use of fatty acids by liver and muscle tissue allows glucose to be conserved for use by the brain. After three days of starvation, the liver produces mass quantities of ketone bodies (acetoacetate and 3-hydroxybutyrate) from acetyl CoA generated from fatty acid oxidation. The brain begins to consume acetoacetate in place of glucose, thereby lowering the need for blood glucose even further. After several weeks of starvation, ketone bodies become the major fuel of the brain, markedly diminishing the need for glucose. Hence, less protein (and therefore less muscle tissue) is degraded than in the earlier stages of starvation. Migratory birds fuel storage system is another example of the biological value of triacylglycerols, which hold six times the energy of glycogen. The birds have large fat deposits under their skin, as well as in their abdomens, muscle and livers. These deposits are so quickly and efficiently mobilized that long migratory flights are possible with little protein degradation.

193. It can be inferred form the passage that glucagon: A. B. C. D. increases the concentration of glucose in the blood. increases the concentration of triacylglycerol in the blood. decreases the concentration of glucose in the blood. decreases the concentration of insulin in the blood.

194. Glycogen is localized mainly in muscle and liver tissue. If it were possible for a migratory bird to use glycogen instead of triacylglycerol as its main stored fuel for a long flight, which of the following would most likely occur? A. B. The bird would use glucose as its main metabolic fuel during migratory flight. The bird would not be able to migrate, because it would have to carry too much weight in fuel for the flight. The birds brain cells would use acetoacetate as their primary fuel during migratory flight. The bird would not be able to complete the migratory flight because it would lack a ready supply of glucose.

C. D.

195. Which of the following is LEAST likely to occur during starvation? A. B. C. D. Gluconeogenesis is promoted. Insulin secretion is inhibited. Triacylglycerol is hydrolyzed. Protein is synthesized.

192. During periods of starvation, the human body can generate glucose from: I. II. III. IV. A. B. C. D. glycogen. the glycerol portion of triacylglycerol. the fatty acid portion of triacylglycerol. amino acids.

I and IV only I, II and III only I, II and IV only I, II, III, and IV

GO ON TO THE NEXT PAGE. 72

Full-Length Test 5

06/27/2003

12:18 PM

Page 73

196. Which of the following best explains why, after several weeks of starvation, acetyl CoA is used almost exclusively to produce ketone bodies? A. Gluconeogenesis depletes the supply of oxaloacetate, which is essential for the entry of acetyl CoA into the Krebs cycle. Starvation leads to the selective production of the enzyme that catalyzes the formation of ketone bodies. Acetyl CoA cannot be converted into pyruvate; therefore, it cannot enter the gluconeogenic pathway. Triacylglycerols are mobilized from adipose tissue.

198. Which of the following is most likely to occur immediately following a meal? A. B. C. D. -oxidation of fatty acids Inhibition of glucagon secretion Increased cellular fermentation Gluconeogenesis

B.

C.

D.

197. Why does the brain use glucose rather than fatty acids as its major metabolic fuel during the early stages of starvation? A. Liver and muscle cells selectively utilize fatty acids at the expense of the brains metabolic requirements. The concentration of triacylglycerol in the blood is not high enough to provide for the brains metabolic requirements. Glucose can cross the blood-brain barrier via specific carrier molecules, while fatty acids cannot. Fatty acids are more easily oxidized in peripheral tissues than in the central nervous system.

B.

C.

D.

GO ON TO THE NEXT PAGE. 73

Full-Length Test 5

06/27/2003

12:18 PM

Page 74

Passage X (Questions 199203) Bimolecular nucleophilic substitution (SN2) reactions play an important role in a number of organic syntheses. SN2 reactions are most successful when carried out in polar aprotic solvents rather than polar protic solvents. However, since polar aprotic solvents often possess high boiling points, they are difficult to remove at the end of the reaction. Nonpolar aprotic solvents such as hydrocarbons possess low boiling points and so would seem a favorable alternative to polar aprotic solvents; however, they do not dissolve ionic compoundsat least, not until the introduction of phase-transfer catalysts. Phase-transfer catalysis (Figure 1) involves the use of two immiscible phasesan organic phase and an aqueous phase. The addition of a phase-transfer catalyst (Q+X) ensures that the nucleophile (Nu) dissolves in the lowboiling organic phase in the form of an ion pair (Q+Nu). The nucleophile dissolves because the cation provided by the catalyst, Q+, is lipophilicit prefers the organic phase to the aqueous phase. As a result, the nucleophile can react with the substrate (RX) to form the SN2 product (RNu), and can be easily removed from the solvent while the original phase-transfer catalyst is transported back to the aqueous layer. CH3(CH2)7Cl (in decane) 1-Chlorooctane

R4N+Br NaCN (aq) 105C

CH3(CH2)7CN Nonanenitrile

Reaction 1 The use of phase-transfer catalysts is not restricted to nucleophilic substitution reactions; they can also be used in oxidation and elimination reactions: Phase C6H5CH2OH (in benzene) + KMnO4(aq) transfer catalysis C6H5COOH

Reaction 2 Phase C6H5CH2CH2Br (in CH2Cl2) + NaOH(aq) transfer catalysis C6H5CH CH2

Reaction 3

Aqueous Phase

NuQ+ X+

XQ+ Nu+

199. What is the correct IUPAC name of the reactant in Reaction 3? A. B. C. D. (2-Bromoethyl) benzene 1-Bromo-2-ethylbenzene 1-Bromo-2-benzylethane 2-Bromo-1-ethylbenzene

Organic Phase

Q+ X+

Q+ Nu+

RNu

RX

Figure 1 Phase-transfer catalysis Phase-transfer catalysts are usually quaternary ammonium salts. An example of the use of this catalyst is the conversion of 1-chlorooctane to nonanenitrile (Reaction 1). 200. Which of the following is NOT a quaternary ammonium salt? A. B. C. D. (CH3)4N+Br C6H5CH2N+(CH2CH3)3Cl CH3NH3+Cl [CH3(CH2)6CH2]3N+CH3Cl

GO ON TO THE NEXT PAGE. 74

Full-Length Test 5

06/27/2003

12:18 PM

Page 75

201. Which of the following represents the transition state formed in Reaction 1?
A. H CN H B. H CN H C. H CN H D. H + CN H C Cl (CH2)6CH3 C Cl (CH2)6CH3 C + Cl (CH2)6CH3 C Cl + (CH2)6CH3

203. Which of the following solvents would be most effective in an SN2 reaction?
A. CH3 CH3 C H B. CH3C OH C. O CH3

HOCH2

CH2OH

D.

O S

202. If aqueous sodium bicarbonate and the organic layer obtained in Reaction 2 are vigorously shaken together in a separatory funnel, benzoic acid: A. B. C. D. reacts to form a sodium salt, but remains in the organic layer. reacts to form a sodium salt and moves to the aqueous layer. doesn't form a sodium salt and remains in the organic layer. doesn't form a sodium salt, but moves to the aqueous layer.

GO ON TO THE NEXT PAGE. 75

Full-Length Test 5

06/27/2003

12:18 PM

Page 76

Passage XI (Questions 204209) Isomers are defined as compounds with the same molecular formula but different structures. The figure below shows the isomer of three different compounds.
Cl Cl H H H H Br CH 3 CH 3 H H H H H H Cl CH 3

205. Compound B exists in equilibrium with the following isomer:


H H H H Compound E H H CH3 H H H Cl H

Br

Compound A I CH 3CH 2 H

Compound B

Which isomer will be favored in the equilibrium? A. Compound B, because nonbonded strain. Compound B, because angle strain. Compound E, because nonbonded strain. Compound E, because angle strain. this molecule has less this molecule has less this molecule has less this molecule has less

CH 2CH 3 H Compound C

B. C. D.

When compound C is treated with hydrogen bromide followed by heating in the presence of potassium hydroxide in ethanol, compound D is formed (Reaction 1).
H CH3CH2 H CH2CH3 H HBr CH3CH2 H CH3CH2 H Br KOH/Ethanol H CH2CH3 Compound D Compound C CH2CH3 H

206. Compound C and Compound D are classed as: A. B. C. D. cis/trans isomers. configurational isomers. meso compounds. enantiomers.

Reaction 1 204. The formation of Compound D from Compound C occurs by: A. B. C. D. nucleophilic substitution. free-radical addition. electrophilic addition followed by elimination. nucleophilic addition followed by elimination.

GO ON TO THE NEXT PAGE. 76

Full-Length Test 5

06/27/2003

12:18 PM

Page 77

207. Which of the following conformers of compound A is lowest in energy?


A. CH3 Cl Br

208. What is hybridization of carbon I in compound C? A. B. C. D. sp3 sp2 sp There is no hybridization between the s and p orbitals.

Cl B.

Br

CH3

Cl Cl CH3 CH3 Br C. CH3 CH3 Cl D. CH3 CH3 Cl Br Cl Br Br Cl

Br

209. What would be the effect on Reaction 1 if Compound C was reacted with HBr in the presence of UV light? A. B. C. D. Compound C would undergo free-radical polymerization. An equal mixture of Compound C and Compound D would be formed. Compound C would not react with HBr. There would be no effect on the reaction.

Br

GO ON TO THE NEXT PAGE. 77

Full-Length Test 5

06/27/2003

12:18 PM

Page 78

Questions 210 through 214 are NOT based on a descriptive passage.


210. When a person inhales fresh air: A. B. C. D. oxygen diffuses from the alveoli into the pulmonary blood. oxygen diffuses from the pulmonary blood into the alveoli. oxygen dissociates from hemoglobin in the pulmonary blood. oxygen dissociates from hemoglobin in the alveoli.

213. Parathyroid hormone, which is an antagonist of calcitonin, stimulates activity in: A. B. C. D. osteoclasts, which dissolve bone. osteoblasts, which form bone. chondrocytes, which secrete cartilage. the kidneys, which excrete vitamin D.

214. Aldosterone is the adrenal hormone responsible for the reabsorption of sodium ions from the distal tubule and collecting duct and the secretion of potassium ions into them. Which of the following symptoms would be present in a patient with Addison's disease, which is the result of adrenocortical deficiency? A. High blood pressure and low [K+] in the extracellular fluid. Low blood pressure and low [K+] in the extracellular fluid. Low blood pressure and high [K+] in the extracellular fluid. No change in either blood pressure or [K+] in the extracellular fluid.

211. Which of the following is NOT a function of bile? A. B. C. D. Emulsification of fat particles into minute particles that are susceptible to the actions of lipases Neutralization of acid emptied into the large intestine from the stomach Transport of the end-products of fat digestion through the intestinal mucosal membrane Excretion of bilirubin, the end-product of hemoglobin destruction
1H

B. C. D.

212. How many peaks would be observed in the spectrum of C(CH3)4? A. B. C. D. 1 2 4 12

NMR

STOP. IF YOU FINISH BEFORE TIME IS CALLED, CHECK YOUR WORK. YOU MAY GO BACK TO ANY QUESTION IN THIS SECTION ONLY.

78

Full-Length Test 5

06/27/2003

12:18 PM

Page 79

Verbal Reasoning Passages Copyright Permissions


From The Deep Roots of Our Decline, by John Attarian. The Wall Street Journal. Copyright 1994. From Duke Ellington, by James Lincoln Collier. Copyright 1987. From The Illusion of Peace: International Relations in Europe, 19181933, by Sally Marks. Copyright 1976. From Sex, Art and American Culture, by Camille Paglia. Copyright 1992. From Citizen Welles, by Frank Brady. Copyright 1989. From The Dreaming Brain, by J. Allan Hobson. Copyright 1989 by Basic Books, Inc. From Say the Right ThingOr Else; Attack Ideas Not People, by Judith Martin and Gunther Stent. Copyright 1991 by The New York Times Company. From Strategies of Containment, by John Lewis Gaddis. Copyright 1982 by Oxford University Press. From The Human Evolution Coloring Book, by Adrienne L. Zihlman. Copyright 1982 by Coloring Concepts, Inc.

79

Full-Length Test 5

06/27/2003

12:18 PM

Page 80

Full-Length Test 5

06/27/2003

12:18 PM

Page 81

Potrebbero piacerti anche